Tag Archives: ChuyenDe

Đa thức bất khả quy

ĐA THỨC BẤT KHẢ QUY

(Thầy Vương Trung Dũng  giáo viên trường PTNK TP Hồ Chí Minh)

1. Giới thiệu sơ lược 

Đa thức bất khả qui là một vấn đề kinh điển trong đa thức nói riêng và trong toán học nói chung. Các bài toán về đa thức bất khả qui cũng thường xuyên xuất hiện trong các kì thi Olympic về toán. Người ta quan tâm nhiều nhất về tính bất khả qui của một đa thức trên vành $\mathbb{Z}[x]$ và $\mathbb{Q}[x]$. Có nhiều cách để kiểm tra tính bất khả qui của một đa thức loại này chẳng hạn như dùng trực tiếp định nghĩa hoặc dùng các tiêu chuẩn như tiêu chuẩn Eisenstein, tiêu chuẩn Perron, tiêu chuẩn Cohn, tiêu chuẩn Dumas… tuy nhiên bài viết này chỉ đề cập đến hai phương pháp thường được sử dụng nhất là sử dụng trực tiếp định nghĩa và tiêu chuẩn Eisenstein và các dạng mở rộng của nó cùng với đó là một kĩ thuật tối quan trọng là rút gọn theo một modulo nguyên tố $p$. Các tiêu chuẩn khác hi vọng sẽ có dịp trình bày trong một bài viết khác.

Trong tài liệu này ta qui ước $\mathbb{Z}_p=\mathbb{Z}/p\mathbb{Z}$ và $\mathbb{K}$ là một trong các tập $\mathbb{Z},\ \mathbb{Q},\ \mathbb{R}, \ \mathbb{Z}_p$. Khi đó, $ \mathbb{K}[x]$ (tương ứng $ \mathbb{K}[x,y]$) là các vành đa thức một biến (tương ứng 2 biến) có hệ số trong $ \mathbb{K}$.

Định nghĩa 1.1:  Đa thức $P(x)$ trong vành $\mathbb{K}[x]$ được gọi là khả qui trên $\mathbb{K}$ nếu $P(x)=f(x).g(x)$ trong đó $f(x), g(x)$ là các đa thức không khả nghịch trong $\mathbb{K}[x]$. Đa thức $P(x)$ được gọi là bất khả qui nếu $P(x)$ không khả nghịch và không khả qui.

Nói riêng, khi $\mathbb{K}$ là một trường thì một đa thức $P(x) \in \mathbb{K}[x]$ có bậc dương được gọi là khả qui trên $\mathbb{K}$ nếu có thể phân tích được thành tích hai đa thức có bậc dương trong $\mathbb{K}[x]$, ngược lại $P(x)$ được gọi là bất khả qui trên $\mathbb{K}$.

Định lí Gauss 1.1: Các vành đa thức

  • $\mathbb{R}[x], \ \mathbb{C}[x],\ \mathbb{Q}[x], \ \mathbb{Z}_p[x]$
  •  $\mathbb{Z}[x], \ \mathbb{Z}[x,y], \ \mathbb{Q}[x,y]…$

là có sự phân tích duy nhất thành các nhân tử bất khả qui và sự phân tích này là duy nhất. Nói riêng các khái niệm về đa thức bất khả qui, ước chung lớn nhất, bội chung nhỏ nhất vẫn còn đúng trên các vành này.

Lưu ý: Trong trường hợp $1$ ở trên là các đa thức có hệ số trên trường nên trên đó thuật toán Euclid hay định lí Bezout vẫn còn đúng nhưng trường hợp $2$ thì không.

2. Tính bất khả qui trên $\mathbb{C}[x]$ và $\mathbb{R}[x]$

Định lí 2.1: Mọi đa thức có bậc lớn hơn 1 đều khả qui trên $\mathbb{C}[x]$.

Chứng minh

Giả sử $degP>1$. Ta thừa nhận định lí cơ bản của đại số “Mọi đa thức $P(x) \in \mathbb{C}[x]$ có bậc lớn hơn 1 đều có ít nhất một nghiệm trên $\mathbb{C}$”. Khi đó $P(x)$ có nghiệm $x_0 \in \mathbb{C}$ nên theo Định lí Bezout $$P(x)=(x-x_0)Q(x),$$

trong đó $deg\ge 1$ nên $P(x)$ khả qui trên $\mathbb{C}[x].$

Định lí 2.2: Mọi đa thức có hệ số thực bậc lớn hơn 2 đều khả qui trên $\mathbb{R}[x]$. Nói riêng một đa thức là bất khả qui trên $\mathbb{R}[x]$ khi và chỉ khi nó là đa thức bậc nhất hoặc bậc 2 vô nghiệm.

Chứng minh

Giả sử $P \in \mathbb{R}[x]$ và $deg P >2$.

  • Nếu $\deg P$ lẻ thì $P$ có ít nhất một nghiệm thực nên nó khả qui.
  • Nếu $\deg P$ chẵn thì $P$ có một nghiệm phức $\alpha$, khi đó $\overline{\alpha}$ cũng là nghiệm của $P$ và do đó $P(x)=(x-\alpha)(x-\overline{\alpha})Q(x)$ là khả qui.

3. Tính bất khả qui trên $\mathbb{Z}[x]$ và $ \mathbb{Q}[x]$

Qua Định lí 2.1 và Định lí 2.2 ta thấy nếu $\mathbb{K}=\mathbb{C},\ \mathbb{R}$ thì tính bất khả quy là đơn giản nên ta quan tâm đến trường hợp $\mathbb{K}=\mathbb{Z}, \ \mathbb{Q}.$ Thật may mắn là bổ đề Gauss mà ta trình bày sau đây sẽ cho ta một sự tương ứng về tính bất khả qui của một đa thức hệ số nguyên trên $\mathbb{Z}[x]$ và $\mathbb{Q}[x]$.

Định nghĩa 3.1: Cho đa thức $P(x)=a_nx^n+a_{n-1}x^{n-1}+…+a_1x+a_0 \in \mathbb{Z}[x]$, đa thức $P$ được gọi là nguyên bản nếu $gcd(a_n,…,a_0)=1$

Mệnh đề 3.1: Tích của hai đa thức nguyên bản là một đa thức nguyên bản.

Mệnh đề 3.2: Mọi đa thức $P \in \mathbb{Q}[x]$ đều viết được dưới dạng $P=cP_0(x)$, trong đó $P_0$ là một đa thức nguyên bản và $c_0 \in \mathbb{Q}.$

Định lí 3.1 (Bổ đề Gauss):  Một đa thức hệ số nguyên, có bậc dương bất khả qui trong $\mathbb{Q}[x]$ khi và chỉ khi nó bất khả qui trong $\mathbb{Z}[x]$.

Chứng minh

Hiển nhiên nếu $P(x)$ bất khả qui trên $\mathbb{Q}[x]$ sẽ bất khả qui trên $\mathbb{Z}[x]$.

Ngược lại giả sử $P(x)$ bất khả qui trên $\mathbb{Z}[x]$ mà

$P(x)=P_1(x)P_2(x)$, với $P_1, P_2 \in \mathbb{Q}[x]$ và $1\le deg \ P_1, degP_2 \le deg\ P$.

Khi đó ta viết lại $P_1=\dfrac{a_1}{b_1}Q_1(x), P_2=\dfrac{a_2}{b_2}Q_2(x)$, với $(a_i,b_i)=1$ và $Q_i$ nguyên bản, $i \in \{1,2\}$.

Suy ra $P(x)=\dfrac{a_1a_2}{b_1b_2}Q_1(x)Q_2(x)=\dfrac{p}{q}Q_1(x)Q_2(x),$ trong đó $p=a_1b_1, q=a_2b_2$ và $ (p,q)=1.$

Do $P\in \mathbb{Z}[x]$ nên các hệ số của $Q_1(x)Q_2(x)$ phải chia hết cho $q$ điều này trái với tính nguyên bản của $Q_1(x)Q_2(x)$.

Từ đó ta có điều phải chứng minh.

Định lí 3.2: Cho $P(x)=a_nx^n+a_{n-1}x^{n-1}+…+a_1x+a_0 \in \mathbb{Z}[x]$. Giả sử $P$ có nghiệm hữu tỉ $x=\dfrac{p}{q}$ với $(p,q)=1$. Khi đó, $p$ là ước của $a_0$ còn $q$ là ước của $a_n.$ Nói riêng, mọi nghiệm hữu tỉ của một đa thức monic (đơn khởi, hệ số của bậc cao nhất bằng $\pm1$) với hệ số nguyên đều là nghiệm nguyên.

Chứng minh

Giả sử $P(x)$ có nghiệm hữu tỉ $\dfrac{p}{q},$ với $ (p,q)=1$. Khi đó $$a_n(\dfrac{p}{q})^n+…+a_1. \dfrac{p}{q}+a_0=0,$$

qui đồng mẫu số ta được $$a_np^n+…+a_0q^n=0.$$

Vì vế phải chia hết cho $p$ nên vế trái chia hết cho $p$, từ đó suy ra $a_0q^n$ chia hết cho $p$, lại có $(q^n,p)=1$ nên $a_0$ chia hết cho $p$. Lập luận tương tự ta được $a_n$ chia hết cho $q$.

Đinh lí 3.3: Cho $P \in \mathbb{Q}[x]$ có bậc 2 hoặc 3. Khi đó, $P(x)$ là bất khả qui khi và khi khi $P(x)$ không có nghiệm hữu tỉ.

Chứng minh

Hiển nhiên nếu $P$ có nghiệm hữu tỉ thì nó khả qui.

Đảo lại, nếu $P$ khả qui thì $P$ phân tích được thành tích của hai đa thức hữu tỉ.

Điều kiện bậc của $P$ bằng 2 hoặc 3 chứng tỏ một trong hai nhân tử trên phải có bậc 1.

Từ đó suy ra $f$ có nghiệm hữu tỉ.

Lưu ý: Định lí trên vẫn còn đúng nếu ta thay $\mathbb{Q}$ bởi một trường $\mathbb{K}$ bất kì. Tức là, đa thức $f \in \mathbb{K}[x]$ với bậc bằng 2 hoặc 3 là bất khả qui nếu và chỉ nếu nó không có nghiệm trong $\mathbb{K}.$

Dưới đây là một số ví dụ

Ví dụ 3.1 (Định lí Schur): Cho các số nguyên phân biệt $a_1, a_2,…,a_n$. Khi đó đa thức

a) $f(x)=(x-a_1)(x-a_2)…(x-a_n)-1$ là bất khả qui trên $\mathbb{Q}[x]$.

b) $f(x)=(x-a_1)(x-a_2)…(x-a_n)+1$ là bất khả qui trên $\mathbb{Q}[x]$ ngoại trừ các trường hợp

  • $(x-a)(x-a-2)+1=(x-a-1)^2$,
  • $(x-a)(x-a-1)(x-a-2)(x-a-3)+1=[(x-a-1)(x-a-2)+1]^2.$
Giải

a) Giả sử $(x-a_1)(x-a_2)…(x-a_n)-1=g(x)h(x)$, với $1 \le deg \ f, \deg \ g \le n-1$ và $g, h \in \mathbb{Z}[x].$

Ta có $g(a_i)h(a_i)=-1$ với mọi $i$, từ đó do $g(a_i), h(a_i)$ là các số nguyên nên $ (g+h)(a_i)=0,$ với mọi $i=1,2,…,n.$

Nhưng vì $deg \ (g+h) \le n-1$ triệt tiêu tại $n$ giá trị phân biệt nên $g \equiv – h$.

Từ đó $$(x-a_1)(x-a_2)…(x-a_n)-1=-(g(x))^2.$$

Đẳng thức trên là vô lí vì hệ số cao nhất ở hai vế trái dấu.

b) Lập luận hoàn toàn như trên, giả sử $f(x)$ là khả qui, bằng một phép đổi biến đơn giản ta hoàn toàn có thể viết lại $f$ dưới dạng $$f(x)=x(x-a_1)(x-a_2)…(x-a_{n-1})+1=g(x).h(x),$$ trong đó $0<a_1<a_2<…<a_{n-1}$ và $1 \le g, h \in \mathbb{Z}, \deg(g), \deg(h) \le n-1$.

Từ đẳng thức $g(a_i)h(a_i)=1$ ta suy ra $g(a_i)=h(a_i)= \pm 1$ với mọi $i$ và đẳng thức này xảy ra tại $n$ giá trị phân biệt. Điều đó dẫn dến $g(x)=h(x)$ và ta có $f(x)=g^2(x)$.

Nói cách khác, $\deg(f)=n$ là một số chẵn. Khi đó $f(\dfrac{1}{2})=\dfrac{1}{2}(\dfrac{1}{2}-a_1)…(\dfrac{1}{2}-a_{n-1})+1=1-\dfrac{1}{2^n}(2a_1-1) \ldots (2a_{n-1}-1) \le 1-\dfrac{1}{2^n}1.3 \ldots (2n-3) <0$ với mọi $n \ge 6$ (vô lí). Như vậy $n=2$ hoặc $n=4$.

  • Nếu $n=2$ thì $f(\dfrac{1}{2})=1-\dfrac{1}{4}(2a_1-1) \Rightarrow a_1 \le \dfrac{5}{2}$ và do đó $a_1=1, 2$. Giá trị $a_1$ cho ta $f(x)=x(x-1)+1$ là bất khả qui và $a_1=2$ cho ta $f(x)=x(x-2)+1=(x-1)^2$ là khả qui.
  • Nếu $n=4$ thì $f(\dfrac{1}{2})=1-\dfrac{1}{16}(2a_1-1)(2a_2-1)(2a_3-1) \Rightarrow 0 \le \dfrac{1}{16}(2-1)(3-1)(2a_3-1) \Rightarrow a_3 \le \dfrac{19}{6}$. Xét trường hợp $a_1=1, a_2=2, a_3=3$ ta được $$f(x)=x(x-1)(x-2)(x-3)+1=(x^2-3x+1)^2$$ là khả qui.

Bài toán được chứng minh xong.

Ví dụ 3.2: Cho $a_1, a_2,…,a_n$ là các số nguyên dương phân biệt. Chứng minh rằng đa thức $$P(x)=(x-a_1)^2(x-a_2)^2…(x-a_n)^2+1$$ là bất khả qui trên $\mathbb{Z}.$

Giải

Giả sử $P(x)$ là khả qui, tức tồn tại hai đa thức $G(x), H(x) \in \mathbb{Z}[x]$ có bậc không bé hơn 1 sao cho $P(x)=G(x).H(x)$.

Ta có $P(a_i)=G(a_i).H(a_i)$ với $i=1,2,…,n$ nên $G(a_i)=H(a_i)= \pm 1.$ Ta xét các trường hợp

  • Nếu $deg \ G= \ deg \ H$ thì $deg(G-H) \le n-1 \Rightarrow G \equiv H.$ Từ đó $P(x)=(G(x))^2 \Leftrightarrow 1= \Big(G(x)-(x-a_1)…(x-a_n) \Big)\Big(G(x)+(x-a_1)…(x-a_n) \Big)$ , vô lí vì bậc vế phải luôn không nhỏ thua 1.
  • Nếu $degH<deg G$ thì $degH<n$ mà $H(a_i)= \pm1, i=1,2,…,n$ dẫn đến $H$ là đa thức hằng, vô lí.

Vậy $P(x)$ bất khả qui.

4. Rút gọn modulo $p$ nguyên tố

Kĩ thuật rút gọn modulo $p$ nguyên tố là một kĩ thuật tối quan trọng trong việc chứng minh một đa thức là bất khả qui trên $\mathbb{Z}$. Nó đưa các hệ số từ một trường vô hạn các phần tử về một trường hữu hạn các phần từ, từ đó các tính toán của ta có thể được đơn giản hơn.

Định nghĩa 4.1: Cho $P(x)= \sum \limits_{i=0}^n a_ix_i \in \mathbb{Z}[x], a_n \ne 0$ và $p$ là số nguyên tố. Giả sử $p$ không phải là ước của $gcd(a_1,a_2,…,a_n)$. Ta kí hiệu $\overline{P}$ là đa thức nhận được từ $P$ bằng cách rút gọn các hệ số theo modulo $p$ (lúc này $P(x) \in \mathbb{Z}_p[x]$). Khi đó ta gọi $\overline{P}$ là \textit{đa thức rút gọn theo modulo} $p$ của $P.$

Từ định nghĩa trên ta có sự kiện sau là hiển nhiên $$\overline{P+Q}=\overline{P}+\overline{Q}$$

$$\overline{PQ}=\overline{P}. \ \overline{Q}$$

Định nghĩa 4.2: Nếu đa thức rút gọn modulo $p$ của P bất khả qui thì ta nói đa thức $P$ bất khả qui $mod \ p.$

Định lí 4.1: Với mỗi $P(x) \in \mathbb{Z}[x]$, tồn tại các đa thức $P_1(x), P_2(x), …, P_k(x) \in \mathbb{Z}_p[x]$ sao cho $$\overline{P}(x)=P_1(x).P_2(x)…P_k(x),$$

sự phân tích này là duy nhất theo modulo $p$.

Định lí 4.2: Cho $P(x)= \sum \limits_{i=0}^n a_ix_i \in \mathbb{Z}[x], a_n \ne 0$ và $p$ không là ước của $a_n$. Khi đó, nếu $P(x)$ là bất khả qui $mod \ p$ thì $P(x)$ là bất khả qui. Điều ngược lại của định lí nói chung không đúng.

Chứng minh

Giả sử $P(x)=a_nx^n+a_{n-1}x^{n-1}+…+a_1x+a_0 \in \mathbb{Z}[x]$ và $p$ không là ước của $a_n$.

Giả sử $P(x)=f(x).g(x)$ với $f, g \in \mathbb{Z}[x]$ với $deg \ f, g \ge 1$.

Khi đó $\overline{P}=\overline{f}. \overline{g}$. Do $p$ không là ước của $a_n$ nên bậc của các đa thức $P, f, g$ không thay đổi sau khi rút gọn theo modulo $p$.

Điều này chứng tỏ $\overline{P}$ khả qui theo modulo $p$, vô lí. Ta có điều phải chứng minh.

Ngược lại dễ thấy đa thức $P(x)=x^4+1$ bất khả qui trên $\mathbb{Z}[x]$ nhưng khả qui modulo $p$ với mọi số nguyên tố $p$.

Ví dụ 4.1: Chứng minh đa thức $P(x)=x^5+4x^4+2x^3+5x^2-7$ là bất khả qui.

Giải
  •  Rút gọn theo modulo 2 ta được $\overline{P}(x)=x^5+x^2+1.$
  •  Giả sử $\overline{P}(x)=f(x). g(x)$, với $f, g \in \mathbb{Z}_2$.
  •  Nếu $deg \ f=1$ hoặc $deg \ g=1$ dễ thấy là vô lí vì $\overline{P}$ không có nghiệm trong $\mathbb{Z}_2$.
  •  Suy ra $\overline{P}(x)=(x^2+ax+b)(x^3+cx^2+dx+e)$, với $a,b,c,d,e \in \mathbb{Z}_2$. Đồng nhất hệ số hai vế ta được điều vô lí. Từ đó suy ra điều phải chứng minh.

Ta có thể liệt kê ra các đa thức bất khả qui modulo 2 trong một số trường hợp bậc nhỏ như sau

  • Trường hợp $n=1$ gồm các đa thức: $x, x+1$.
  •  Trường hợp $n=2$ chỉ gồm một đa thức: $1+x+x^2$.
  •  Trường hợp $n=3$ gồm các đa thức: $1+x+x^3, 1+x^2+x^3$.
  •  Trường hợp $n=4$ gồm các đa thức: $1+x+x^4, 1+x+x^2+x^3+x^4$.
  •  Trường hợp $n=5$ gồm các đa thức:

$1+x+x^2+x^4+x^5$,

$1+x+x^3+x^4+x^5$,

$1+x^2+x^3+x^4+x^5$,

$1+x+x^2+x^3+x^5$,

$1+x^3+x^5, 1+x^2+x^5.$

5. Tiêu chuẩn Eisenstein và một số dạng mở rộng

Khi kiểm tra đa tính bất khả qui của một đa thức trên $\mathbb{Z}[x]$ tiêu chuẩn Eisenstein cung cấp cho ta một công cụ hiệu quả.

Định lí 5.1 (Tiêu chuẩn Eisenstein): Cho đa thức $P(x)= \sum \limits_{i=0}^na_ix^i \in \mathbb{Z}[x], a_n \ne 0$. Khi đó nếu tồn tại số nguyên tố $p$ thỏa đồng thời các điều kiện

  • $p$ không là ước của $a_n$;
  • $p$ là ước của $a_i$ với mọi $i\in \{1,2,…,n-1\}$;
  • $p^2$ không là ước của $a_0.$

Khi đó $P(x)$ là đa thức bất khả qui trên $\mathbb{Q}[x].$

Chứng minh

Có rất nhiều cách chứng minh cho tiêu chuẩn Eisenstein, ở đây ta sẽ trình bày chứng minh bằng cách rút gọn theo một modulo $p$ nguyên tố bất kì.

Giả sử $f$ khả qui, tức $f(x)=g(x).h(x)$, với $f, g \in \mathbb{Z}[x]$ và $deg \ f, g \ge 1.$ Rút gọn theo modulo $p$ nguyên tố ta được đẳng thức trong $\mathbb{Z}_p[x]$ dưới dạng $$\overline{f}=\overline{g}. \overline{h}.$$

Từ điều kiện $p$ là ước của $a_0, …, a_{n-1}$ nhưng không là ước của $a_n$ ta suy ra $\overline{f}=\overline{a_n}x^n.$

Từ đó suy ra $\overline{g}= \overline{b_k}x^k, \overline{h}=\overline{b_m}x^m$. Điều này có nghĩa là $\overline{b_0} \equiv…\equiv \overline{b_{k-1}} \equiv \overline{c_0} \equiv… \equiv \overline{c_{m-1}} \equiv 0 \ mod(p)$. Nhưng khi đó $a_0=b_0c_0 \equiv0 \ (mod \ p^2)$ (vô lí). Từ đó ta có điều phải chứng minh. $\square$

Ví dụ 5.1: Chứng minh đa thức $P(x)=x^4-x^2+2x+1$ bất khả qui trên $\mathbb{Z}$.

Giải

Đặt $Q(x)=P(x+1)=x^4+3x^3+3x^2+3x+3$. Khi đó theo tiêu chuẩn Eisenstein với $p=3$ ta có điều phải chứng minh.

Định lí 5.2 (Dạng mở rộng thứ nhất của tiêu chuẩn Eisenstein):

Cho $f(x)=a_nx^n+a_{n-1}x^{n-1}+…+a_1x+a_0 \in \mathbb{Z}[x]$. Giả sử tồn tại số nguyên tố $p$ thỏa mãn với một số tự nhiên $k \le n$ nào đó mà

  • p không là ước của $a_k$;
  • $p$ là ước của $a_0, …, a_{k-1}$;
  • $p^2$ không là ước của $a_0$.

Thế thì $f(x)$ có một nhân tử bất khả qui bậc $ \ge k$ ( và do đó nếu không bất khả qui sẽ có một nhân tử bậc $\le n-k$)

Chứng minh: Bạn đọc có thể tự chứng minh như trong trường hợp nguyên bản của định lí.

Ví dụ 5.2: Chứng minh đa thức $f(x)=x^{101}+101x^{100}+102$ là bất khả qui.

Giải

Áp dụng tiêu chuẩn Eisenstein mở rộng cho $p=2, k=100$ ta thấy, nếu $f$ là khả qui thì nó phải có một nhân tử bậc 1 và do đó $f$ phải có nghiệm hữu tỉ. Nói riêng vì hệ số bậc cao nhất bằng 1 nên nghiệm hữu tỉ này phải là nghiệm nguyên. Dễ thấy điều này là không xảy ra. bài toán được chứng minh xong.

 

Định lí 5.3 (Dạng mở rộng thứ hai của tiêu chuẩn Eisenstein):

Cho $f(x)=a_nx^n+a_{n-1}x^{n-1}+…+a_1x+a_0 \in \mathbb{Z}[x]$. Giả sử tồn tại số nguyên tố $p$ thỏa mãn với một số tự nhiên $k \le n$ nào đó mà

  • p không là ước của $a_n$;
  • $p$ là ước của $a_0, …, a_{n-1}$;
  • $p^2$ không là ước của $a_k$.

Thế thì, hoặc $f(x)$ là bất khả qui, hoặc $f$ có một nhân tử bất khả qui bậc $ \le k.$

Tương tự như trên, chứng minh được dành cho bạn đọc.

6. Các bài toán áp dụng

Bài tập 6.1 (IMO 1993): Cho số tự nhiên $n>1$. Chứng minh đa thức $f(x)=x^n+5x^{n-1}+3$ là bất khả qui trên $\mathbb{Z}[x]$.

Giải

Áp dụng dạng mở rộng thứ nhất của tiêu chuẩn Eisenstein với $p=3, k=n-1$ ta có điều phải chứng minh.

Bài tập 6.2 (China TST 1994): Cho số tự nhiên $n \ge 3$ và hai số nguyên tố $p, q$ phân biệt. Tìm tất cả các số nguyên $a$ sao cho đa thức $P(x)=x^n+ax^{n-1}+pq$ bất khả qui trên $\mathbb{Z}.$

Giải

Nếu $p|a$ hoặc $q|a$ thì theo tiêu chuẩn Eisenstein $P(x)$ là bất khả qui. Xét trường hợp $p,q$ đều không là ước của $a$. Giả sử $P$ khả qui, áp dụng dạng mở rộng thứ nhất của tiêu chuẩn Eisenstein ta suy ra $P(x)$ phải có nhân tử bậc 1 và do đó $P$ có một nghiệm nguyên $x_0$.

Từ đó suy ra $pq=-x_0^{n-1}(x_0+a)$. Vì $n \ge 3$ nên $pq \ \vdots \ x_0^2$ nhưng vì $p \ne q$ nên $x_0=\pm 1.$

Vì $1+a+pq=0$ và $(-1)^n+a(-1)^{n-1}+pq=0$ nên $a=-1-pq$ hoặc $a=1+(-1)^npq.$

Bài tập 6.3 (Rumani TST 1998): Chứng minh rằng đa thức $P(x)=(x^2+x)^{2^n}+1$ là bất khả qui với mọi số tự nhiên $n$.

Giải

Bằng qui nạp ta chỉ ra rút gọn modulo 2 thì đa thức đã cho trở thành $(x^2+x+1)^{2^n}$. Chú ý rằng đa thức $x^2+x+1$ là bất khả qui modulo 2.

Giả sử $P(x)$ khả qui, tức tồn tại hai đa thức $f,h$ đơn khởi với $f, g \in \mathbb{Z}[x], deg \ f,g \ge 1$ sao cho $P(x)=f(x).g(x)$. Khi đó $\overline{f}=(x^2+x+1)^k, \overline{g}=(x^2+x+1)^{2^n-k}$ và $$f(x)=(x^2+x+1)^k+2f_0(x), g(x)=(x^2+x+1)^{2^n-k}+2g_0(x),$$

với $f_0, g_0 \in \mathbb{Z}[x]$.

Gọi $j$ là một căn bậc 3 khác 1 của đơn vị. Thay $j$ vào đẳng thức $P(x)=f(x).h(x)$ ta được $$P(j)=g(j).h(j) \Leftrightarrow 2=4f_0(j)g_0(j).$$

Từ đó suy ra $f_0(j).g_0(j)=\dfrac{1}{2}$.

Do $f_0(j)g_0(j)$ luôn viết được dưới dạng $aj+b; a, b \in \mathbb{Z}$ và đẳng thức này không thể xảy ra. Ta có điều phải chứng minh.

Một số bài toán tương tự như sau:

Bài 1: Với $n \ge 1$ là số tự nhiên, chứng minh các đa thức sau là bất khả qui trên $\mathbb{Z}$

a) $P(x)=(x^3+x)^{2^n}-3$

b) $P(x)=(x^2+ax)^{2^n}+1$ với $ a \in \mathbb{Z}$

Bài 2: Cho $p$ là một số nguyên tố có dạng $4k+3$. Chứng minh rằng với mọi số nguyên dương $n$ đa thức $P(x)=(x^2+1)^n+p$ bất khả qui trên $\mathbb{Z}[x]$.

Bài 3: Cho $p$ là một số nguyên tố và $a$ là một số nguyên không chia hết cho $p$. Chứng minh đa thức $P(x)=x^p-x+a$ bất khả qui trên $\mathbb{Z}[x].$

Bài tập 6.4 (Japan 99): Chứng minh rằng đa thức $f(x)=(x^2+1^2)(x^2+2^2)…(x^2+n^2)+1$ là bất khả qui trên $\mathbb{Z}$

Giải

Giả sử $n \ge 2$ vì trường hợp $n=1$ là tầm thường và giả sử $f(x)=g(x).h(x)$ với $ f, g \in \mathbb{Z}[x]$ và $1 \le deg \ f, g \le n-1$. Khi đó $$1=f(\pm ki)=g(\pm ki) h(\pm ki).$$

Vì $f, g \in \mathbb{Z}[x]$ nên $g(\pm ki), h(\pm ki)$ có dạng $a + bi$ . Từ đó suy ra $$1=g(ki)h(ki)=1.1=(-1).(-1)=i.(-i)=(-i).i$$

Như vậy trong tất cả $4$ trường hợp ta đều có $g(ki)=\overline{h(ki)}=h(-ki),$ với $k=1,2,…,n$. Như vậy đa thức $P(x)=g(x)-h(-x)$ có $2n$ nghiệm phân biệt nhưng có bậc nhỏ hơn $2n$ nên là đa thức 0 và do đó $g(x)=h(-x)$. Suy ra $\deg(g)=\deg(h)=n $.

Vì $f$ đơn khởi nên $g,h$ cũng đơn khởi. Khi đó $g^2-h^2$ có bậc không quá $2n-1$ nhưng lại có ít nhất $2n$ nghiệm $ki$, với $i \in \{-n, -n-1,…,-1,1,…,n\}$ nên $g^2=h^2$.

Nhưng dễ thấy $g=-h$ không xảy ra do đó $g\equiv h$. Khi đó $$f(x)=g(x)^2 \Rightarrow f(0)=(g(0))^2=(n!)^2+1,$$ vô lí. Bài toán được chứng minh xong.

Ta có bài toán tổng quát hơn là: Cho $p$ là một số nguyên tố. Chứng minh rằng với mỗi số tự nhiên $n$ đa thức $$P(x)=(x^p+1^2)(x^p+2^2)…(x^p+n^2)+1$$ bất khả qui trên $\mathbb{Z}[x].$

Bài tập 6.5: Cho $m, n, a$ là các số nguyên dương và số nguyên tố $p$ thỏa mãn $p<a-1$. Chứng minh rằng đa thức $P(x)=x^m(x-a)^n+p$ bất khả qui trên $\mathbb{Z}$.

Giải

Giả sử $P(x)$ khả qui và $P(x)=G(x).H(x)$, với $G, H \in \mathbb{Z}[x]$. Vì $P(0)=p$ nên $|G(0)|=1$ hoặc $|H(0)|=1$. Không mất tổng quát ta giả sử $G(x)=x^k+a_{k-1}x^{k-1}+…+a_0$ và $|G(0)|=1, m+n-1 \ge k \ge 1.

Gọi $x_1,…,x_k$ là nghiệm của $G(x)$. Ta viết $G(x)$ dưới dạng $G(x)=(x-x_1)…(x-x_k)$, dẫn đến $|x_1…x_k|=1$ và $P(x_i)=0 \Leftrightarrow x_i^m(x_i-a)^n=-p$.

Cho $i=\overline{1,k}$ và nhân các vế của đẳng thức lại ta được $$ |G(x)|^n=p^k \ \text{nên} \ |G(a)|^n=p^k.$$

Mặt khác $P(a)=G(a).H(a)=p$ nên ta suy ra $|G(a)|=p$. Do đó $|G(a)-G(0)|=p \pm 1$ chia hết cho $a$.

Vì thế nên $p-1 \ge a$ hoặc $p+1 \ge a$ (mâu thuẫn với giả thiết $p<a-1$).

Vậy $P(x)$ là bất khả qui.

Bài tập 6.6 (Rumani 1999): Cho số nguyên $a$ và số nguyên dương $n$ và $p$ là một số nguyên tố thoả $p>|a|+1$. Chứng minh rằng đa thức $P(x)=x^n+ax+p$ bất khả qui trên $\mathbb{Z}[x]$.

Giải

Giả sử $P(x)=g(x).h(x)$, với $g, h \in \mathbb{Z}[x]$ và $1 \le deg f, deg g \le n-1$. Vì $P(0)=p=g(0).h(0)$ nên không mất tổng quát giả sử $g(0)=\pm 1, h(0)= \pm p.$

Khi đó $g(x)=\pm x^m+T(x)\pm 1, T \in \mathbb{Z}[x], deg T \le m-1.$

Gọi $z_1, z_2,\ldots,z_m$ là nghiệm của $g(x)$. Theo định lí Viet $1=|g(0)|=|z_1z_2…z_m|$ nên $|z_i| \le 1, i=1,2,…,m.$

Lại có $0=f(z_i)=z_i^n+az_i+p$ nên $$p=-z_i^n+az_i \le |z_i|^n+|a|.|z_i| \le 1+|a|,$$

vô lí. Vậy ta có điều phải chứng minh.

Bài tập 6.7: Cho $p, q$ là hai số nguyên tố lẻ sao cho $q$ không là ước của $p-1$ và gọi $a_1, a_2,…,a_n$ là các số nguyên phân biệt sao cho $q|(a_i-a_j)$ với mọi cặp $(i,j)$. Chứng minh rằng $$P(x)=(x-a_1)(x-a_2)…(x-a_n)-p$$ là bất khả qui trên $\mathbb{Z}[x]$ với mọi $n \ge 2.$

Giải

Giả sử $f$ khả qui trên $\mathbb{Z}[x]$. Khi đó tồn tại $Q(x), R(x) \in \mathbb{Z}[x]$ sao cho $Q(x)R(x)=P(x)$ và $1 \le deg Q(x), \le deg R(x).$ Nói riêng $degQ(x) \le \dfrac{n}{2}$.

Ta có $Q(a_i)=R(a_i)=-p,$ với $1 \le i \le n$ từ đó suy ra $Q(a_i), R(a_i) \in \{-1,1,-p,p \}$ với mọi $1 \le i \le n$. Với mọi hằng số $c$ đa thức $Q(x)-c$ nhận nhiều nhất $degQ(x) \le \dfrac{n}{2}$ nghiệm. Do đó $Q(a_i)$ nhận ít nhất hai giá trị phân biệt (và ít nhất $3$ giá trị phân biệt nếu $degQ(x)< \dfrac{n}{2}$).

Vì $q|(a_i-a_j)$ nên $q|(Q(a_i)-Q(a_j)$. Để ý rằng $q$ là số nguyên tố lẻ nên $Q(a_i)$ không thể nhận hai giá trị $1$ và $-1$ (vì nếu ngược lại thì $q|1-(-1)=2$). Tương tự $Q$ cũng không thể nhận hai giá trị là $p -p$ vì khi đó $R(a_i)$ nhận hai giá trị là $1,-1$.

Từ giả thiết $q$ không là ước của $p-1$ suy ra $Q(a_i)$ không thể nhận hai giá trị $1$ và $p$ hoặc $-1$ và $-p$. Do đó $Q(a_i)$ chỉ có thể nhận được nhiều nhất hai giá trị là $1$ và $-p$ hoặc $-1$ và $p$. Giả sử trường hợp đầu tiên xảy ra.

Vì $Q(a_i)$ chỉ nhận hai giá trị nên $degQ(a_i)=\dfrac{n}{2}$ và $Q$ nhận mỗi giá trị $1$ và $-p$ đúng $\dfrac{n}{2}$ lần. Chia tập $(a_i)_{i=1}^n$ thành hai tập $(b_i)_{i=1}^n$ và $(c_i)_{i=1}^n$ sao cho $Q(b_i)=1$ và $Q(c_i)=-p$. Khi đó $$Q(x)=(x-b_1)(x-b_2)…(x-b_{\frac{n}{2}})+1=(x-c_1)(x-c_2)…(x-c_{\frac{n}{2}})-p.$$

Vì ta cũng có $degR(x)=\dfrac{n}{2}$ nên $R(a_i)=-p$ khi $Q(a_i)=1$. Do đó $$R(x)=(x-b_1)(x-b_2)…(x-b_{\frac{n}{2}})-p=(x-c_1)(x-c_2)…(x-c_{\frac{n}{2}})+1.$$

Nhưng khi đó để ý rằng trong đẳng thức thứ nhất cho ta $Q(x)-R(x)=1+p$ còn đẳng thức thứ 2 cho ta $Q(x)-R(x)=-p-1$ điều đó dẫn đến $p=-1$. Vô lí. Bài toán được chứng minh xong.

Bài tập 6.8: Tìm tất cả các cặp số nguyên dương $(m,n)$ sao cho đa thức $$P(x,y)=(x+y)^2(mxy+n)+1$$ khả qui trên $\mathbb{Z}[x,y]$. Khi đó hãy phân tích $f$ thành các nhân tử bất khả qui.

Giải

Đặt $S=x+y$ ta viết lại $f$ dưới dạng $$f(x,S)=S^2(mx(S-x)+n)+1=-mS^2x^2+mS^3x+(nS^2+1).$$

Ta xem $f$ là một tam thức bậc 2 theo biến $x$. Khi đó $f$ khả qui khi và chỉ khi $f$ phân tích được thành tích của hai đa thức bậc nhất. Khi đó biệt thức $$\Delta =m^2S^6+4mS^2(nS^2+1)=mS^2(mS^4+4nS^2+4)$$ là một bình phương.

Dễ thấy điều này xảy ra khi và chỉ khi $m=n^2$, lúc này $$\Delta= (nS(nS^2+2))^2$$ và $f$ có hai nghiệm là $x=\dfrac{-1}{nS}$ hoặc $x=\dfrac{nS^2+1}{nS}$.

Khi đó $$f(x)=(nSx+1)(-nSx+nS^2+1)=(nx^2+nxy+1)(ny^2+nxy+1).$$

 

7. Bài tập tự luyện

Bài 1: Với $n \ge 2$ là một số nguyên và $r=\sqrt[n]{2}$. Chứng minh rằng không tồn tại các số hữu tỷ $a_0, a_1,…,a_{n-1}$ không đồng thời bằng $0$ sao cho $$ a_0+a_1r+a_2r^2+…+a_{n-1}r^{n-1}=0 $$

Bài 2: Tìm số nguyên dương $n$ nhỏ nhất sao cho đa thức $P(x)=x^{n-4}+4n$ có thể phân tích được thành tích của 4 đa thức hệ số nguyên và không là đa thức hằng.

Bài 3: Cho $P(x), Q(x)$ là hai đa thức đơn khởi, bất khả quy trên trường số hữu tỷ. Giả sử $P, Q$ có hai nghiệm tương ứng là $\alpha, \beta$ sao cho $\alpha +\beta$ là số hữu tỷ. Chứng minh $P^2(x)-Q^2(x)$ có nghiệm hữu tỷ.

Bài 4: Chứng minh đa thức $P(x)=(1+x+x^2+…+x^n)^2-x^n$ khả qui trên $\mathbb{Z}[x].$

Bài 5: Chứng minh rằng đa thức $P(x)=x^n+4$ khả qui trên $\mathbb{Z}$ khi và chỉ khi $n$ là bội của $4.$

Bài 6 (IMO Longlist 1989): Cho $n \ge 4$ và các số nguyên phân biệt $a_1,a_2,…,a_n$. Chứng minh đa thức $$P(x)=(x-a_1)(x-a_2)…(x-a_n)-2$$ bất khả qui trên $\mathbb{Q}[x].$

Bài 7 (VMO 2014): Cho $n$ là số nguyên dương. Chứng minh rằng đa thức $P(x)=(x^2-7x+6)^n+13$ không thể biểu diễn được thành tích của $n+1$ đa thức khác hằng với hệ số nguyên.

Bài 8: Chứng minh rằng đa thức $x^n-x-1$ bất khả qui trên $\mathbb{Q}[x]$, với mọi $n \ge 2.$

Bài 9: Cho $n>m>1$ là hai số nguyên lẻ. Chứng minh đa thức $P(x)=x^n+x^m+x+1$ bất khả qui trên $\mathbb{Z}[x]$.

Bài 10: Cho $p$ là số nguyên tố. Chứng minh rằng đa thức $$P(x)=x^{p-1}+2x^{p-2}+…+(p-1)x+p$$ bất khả qui trên $\mathbb{Z}$.

Bài 11: Cho đa thức $P(x)=a_nx^n+a_{n-1}x^{n-1}+…+a_1x+a_0 \in \mathbb{Z}[x], (a_n \ne 0, n \ge 2)$. Chứng minh rằng tồn tại vô số số nguyên tố $k$ sao cho đa thức $P(x)+k$ bất khả qui.

Bài 12: Tìm tất cả các số nguyên $n$ sao cho đa thức $P(x)=x^5-nx-n-2$ là khả qui trên $\mathbb{Z}[x]$.

Bài 13: Cho $p$ là một số nguyên tố và $n$ là một số nguyên nhỏ hơn 4. Chứng minh rằng nếu $a$ là một số nguyên không chia hết cho $p$ thì đa thức $P(x)=ax^n-px^2+px+p^2$ bất khả qui trên $\mathbb{Z}[x].$

Bài 14: Cho $p$ là số nguyên tố. Chứng minh rằng đa thức $P(x)=x^p+(p-1)!$ bất khả qui trên $\mathbb{Z}[x]$.

Bài 15: Tồn tại hay không đa thức $f \in \mathbb{Q}[x]$ sao cho $f(1) \ne -1$ và $x^nf(x)+1$ là khả qui với mọi $n \in \mathbb{N}$.

Bài 16: Cho $a$ là một số nguyên dương và $p \ge 2 $ là một số nguyên tố thỏa mãn $(a,p)=1$. Chứng minh rằng đa thức $P(x)=x^p-mx+a$ bất khả qui trên $\mathbb{Z}[x]$ với $m \equiv \ 1 \ (mod \ p)$.

Bài 17: Cho $p$ là một số nguyên tố lẻ. Chứng minh đa thức $P(x)= \sum \limits_{i=0}^{p-2}(p-1-i)x^i$ bất khả qui trên $\mathbb{Q}[x].$

Bài 18 (Rumani TST 2003): Cho $P(x) \in \mathbb{Z}[x]$ là một đa thức monic bất khả qui trên $\mathbb{Z}[x]$ sao cho $P(0)$ không là số chính phương. Chứng minh rằng $Q(x)=P(x^2)$ cũng bất khả qui trên $\mathbb{Z}[x].$

Bài 19 (China TST 2006): Cho số nguyên $n \ge 2$. Chứng minh rằng tồn tại đa thức $P(x)=x^n+a_{n-1}x^{n-1}+…+a_1x+a_0$ thỏa mãn

a) $a_0, a_1,…,a_{n-1}$ khác 0.

b) $P(x)$ bất khả qui.

c) Với mọi số nguyên $x$ thì $|P(x)|$ không là số nguyên tố.

Bài 20: Biết $f \in \mathbb{Z}[x]$ là một đa thức bất khả qui có bậc lẻ và lớn hơn 3. Giả sử rằng các nghiệm của $P$ đều có modun lớn hơn 1 và $f(0)$ không có ước chính phương. Chứng minh rằng đa thức $g(x)=f(x^3)$ cũng là đa thức bất khả qui.

Bài 21: Cho $f \in \mathbb{Z}[x]$ là một đa thức monic với bậc lớn hơn 1. Giả sử $f(x^n)$ bất khả qui trên $\mathbb{Z}[x]$ với mọi $n \ge 2$. Hỏi $f$ có bất khả qui trên $\mathbb{Z}[x]$ hay không?

Bài 22: Cho $1 \ne f \in \mathbb{Z}[x]$ sao cho có vô hạn số nguyên $a$ thỏa $f(x^2+ax)$ bất khả qui trên $\mathbb{Q}[x]$. Hỏi $f$ có bất khả qui trên $\mathbb{Q}[x]$ hay không?

Bài 23: Cho $f(x) \ne \pm x$ là một đa thức bất khả qui trên $\mathbb{Z}[x]$. Hỏi $f(x.y)$ có bất khả qui trên $\mathbb{Z}[x,y]$ hay không?

Tài liệu tham khảo

[1] Nguyễn Tiến Quang, NXB Giáo dục, Đại số đại cương

[2] Đoàn Duy Cường, 2015, Bài giảng bồi dưỡng giáo viên chuyên toán năm

[3] Nguyễn Chu Gia Vượng,2015,  Đa thức bất khả qui

[4] Exploration-Creativity 2012,  Irreducible polynomials

[5] Yufei Zhao, Integer polynomial 

[6] Dusan Djukic, Polynomials in one variable 

[7] Gabriel D.Carroll, Polynomials 

[8] Victor V.Prasolov, Polynomials

[9] Titu Andresscu, Gabriel Dospinescu, Problems from the book

[10] U298, Mathematical Reflections

[11] https://artofproblemsolving.com/community/c89 

Tổ hợp lặp – Bài toán chia kẹo Euler (Phần 1)

Trong các bài toán đếm ta gặp bài toán sau: Một người vào cửa hang mua dụng cụ học tập để làm thành một món quà gồm viết, sách và tập, người đó chỉ mua tổng cộng 5 món đồ. Biết rằng trong cửa hàng có 5 cây viết giống nhau, 6 sách giống nhau và 10 cuốn tập giống nhau, hỏi có bao nhiêu cách chọn viết, sách tập để làm quà?

Ta thấy rằng số lượng các viết sách và tập đều lớn hơn số cần mua, do đó bài toán chỉ quay lại việc đếm là có bao nhiêu bộ sách viết tập mà tổng số là 5 cái, trong đó mỗi cái có hoặc không có.

Có ba đối tượng là viết, sách và tập, tạ kí hiệu là $A = { V, S, T }$. Một món quà gồm 5 cái, do đó quà có thể là $X = { V, V, V, S, T }$, gồm 3 cây viết và 1 sách, 1 tập, hoặc là tập $Y = { V, V, S, T, T }$, ta thấy các đối tượng $V, T$ là lập lại. Khi đó ta nói tổ hợp $X, Y$ là tổ hợp lặp.

Để định nghĩa rõ hơn ta có định nghĩa sau:

Định nghĩa.  Cho tập $A = { a_1, a_2, \cdots, a_k }$. Một ánh xạ từ $p: A \mapsto \mathbb{N} $, khi đó $P$ được gọi là một multiset của A.

Ví dụ 1. Cho $A = { a, b, c }$. Ánh xạ $p: A \mapsto \mathbb{N}$ như sau: $p(a) = 2, p(b) = 1, p(c) = 1$. Khi đó ta có thể kí hiệu $p$ là $(aabc)$, hay $(baac)$,.., không tính đến thứ tự của các phần tử $a, b, c$.

Đặt $n = p(a_1) + p(a_2)+\cdots +p(a_k)$, bài toán đặt ra là có bao nhiêu ánh xạ $p: A \mapsto \mathbb{N}$ mà $n = p(a_1) + p(a_2)+\cdots +p(a_k)$.

Tiếp theo ví dụ trên, nếu $ p(a) + p(b) + p(c) = 2$ thì có các multiset sau: $(ab), (ac), (bc), (aa), (bb), (cc)$, 6 multiset.

Tính chất. Cho tập $A = { a_1, a_2, \cdots, a_k }$, số ánh xạ $p: A \mapsto \mathbb{N}$ thỏa $p(a_1) + \cdots + p(a_k) = n$ là $C^n_{n+k-1}$

Chứng minh

Mỗi ánh xạ $p$ ta cho tương ứng với một dãy nhị phân độ dài $n+k-1$, trong đó $p(a_1)$ chữ số đầu là 0, tiếp theo là số 1, rồi $p(a_2)$ chữ số $0$,…cuối cùng là $p(a_k)$ chữ số $0$. Ví dụ bộ $VVSTT$ ứng với dãy $0010100$.

Rõ ràng đây là tương ứng 1 – 1, do đó số ánh xạ $p$ bằng số dãy nhị phân, do đó ta chỉ cần đếm số dãy nhị phân.

Ta thấy dãy có $n+k-1$ chữ số trong đó có $k-1$ chữ số $1$, do đó số dãy nhị phân chỉ là số cách chọn vị trí cho $k-1$ chữ số $1$ nên số dãy nhị phân là $C^{k-1}_{n+k-1}$.

Do đó số ánh xạ $p$ là $C^{k-1}_{n+k-1} $

Trở lại bài toán trên, ta thấy số món quà có 5 cái là một tổ hợp lặp chập 5 của sách, viết, tập, do đó số món quà có thể là $C^{2}_{5+2-1} = C^2_6 = 15$.

(Chú ý trong bài toán trên, đảm bảo số mỗi loại sản phẩm có không ít hơn 5 cái).

Bài toán 1. (Chia kẹo Euler). Cho $n$ viên kẹo giống nhau đem chia cho $k$ người, hỏi có bao nhiều cách chia.

Giải

Ta gọi $k$ người là $a_1, a_2, \cdots a_k$, với mỗi cách chia kẹo là một multiset của $A$ mà $p(a_1) + p(a_2)+\cdots +p(a_k) = n$.

Do đó số cách chia kẹo là $C^n_{n+k-1}$.

Bài toán 2. Giải bài toán trên với cách chia sao cho mỗi người có ít nhất một viên.

Giải

Trước hết phát cho mỗi người một viên, thì còn $n-k$ viên kẹo, tiếp tục áp dụng bài toán trên với $n-k$. Khi đó số cách chia là

$C^{k-1}_{n-1}$

Ta có thể giải bài toán trên mà không cần sử dụng bài toán 1 bằng cách xây dựng dãy nhị phân thỏa: $a_1$ chữ số đầu là 0, tiếp theo là số 1, tiếp là $a_2$ chữ số 0, …., cuối cùng là $a_k$ chữ số 0. Dãy này có $k-1$ chữ số 1 đứng giữa $n$ chữ số 0 và không có hai chữ số $1$ nào đứng kề nhau. Khi đó số dãy nhị phân là: $C^{k-1}_{n-1}$.

Phần kế tiếp ta cùng tìm hiểu và giải một số bài toán có thể đưa về bài toán tổ hợp lặp hay bài toán chia kẹo Euler. Các bạn chờ nhé.

Bài toán 1 và 2 có thể phát biểu dưới dạng sau.

Bài toán 3. Cho phương trình $x_1 + x_2 + \cdots + x_k = n$ trong đó $k, n$ là các số nguyên dương.

a. Tìm số nghiệm tự nhiên của phương trình.

b. Tìm số nghiệm nguyên dương của phương trình.

Như bài toán trên ta đã biết, số nghiệm tự nhiên của phương trình là  $C^{k-1}_{n+k-1}$.

Số nghiệm nguyên dương của phương trình là $C^{k-1}_{n-1}$.

(Phần 2)

 

PHƯƠNG PHÁP ĐẾM BẰNG HAI CÁCH – Phần 1

PHƯƠNG PHÁP ĐẾM BẰNG HAI CÁCH

(Dành cho học sinh lớp 10 chuyên toán)

Lời nói đầu
Đếm bằng hai cách là một phương pháp hay gặp trong đời sống, ví dụ bài toán sau: Một công ty nhập vào 3 xe hàng $ A, B, C $ gồm hai loại hàng $ I $ và $ II $. Trong đó xe $ A $ có 3 loại $ I $ và 2 loại $ II $, xe $ B $ có 4 loại $ I $ và 6 loại $ II $, xe $ C $ có 4 loại $ I $ và 6 loại $ II $. Tính số lượng hàng mà công ty nhâp vào. Đây là bài toán khá đơn giản, để giải bài toán ta có thể lập bảng và khi đó ta có thể tính bằng 2 cách như sau: Tính tổng số hàng trên mỗi xe rồi cộng lại; hoặc ta có thể tính tổng số hàng loại $ I $ trên 3 xe,tổng số hàng loại 2 trên 3 xe, rồi sau đó cộng lại.


Trên đây là một ví dụ của tính bằng hai cách, ta có thể tính tổng theo dòng hoặc có thể tính tổng theo cột. Tổng quát hơn ta có công thức đại số sau: $\sum_{i \in I,j \in J}a_{ij}=\sum_{j \in J}(\sum_{j \in J}a_{ij})=\sum_{j \in J}(\sum_{i \in J}a_{ij})$

Trong một số tình huống đề bài yêu cầu đếm số phần tử của một tập hợp mà không quan tâm ta đếm bằng cách nào, khi đó đếm bằng hai cách cho ta cùng một đáp số giống nhau, khi đó ta sẽ thiết lập được một đẳng thức tổ hợp. Một ví dụ đơn giản như đếm số tập con của tập có $ n $ phần tử, ta có thể đếm số tập có $ k $ phần tử với $ k = 0,1,…,n $, lấy tổng ta được $ C^0_n +C^1_n +….+C^n_n $. Nhưng nếu ta đếm bằng cách khác như sau: xét một tập hợp $ A $ bất kì, khi đó phần tử $ i $ có thể thuộc $ A $ hoặc $ i $ không thuộc $ A $, mỗi phần tử có 2 trường hợp, mà có $ n $ phần tử nên số tập $ A $ là $ 2^n $. Từ đó ta có đẳng thức $ C^0_n + C^1_n + …. + C^n_n = 2^n $. Đếm bằng hai cách cho ta một phương pháp để chứng minh đẳng thức liên quan tới hệ số khai triển nhị phân hay các đẳng thức tổ hợp.

Ngoài ra đếm bằng hai cách có thể áp dụng trong các bài toán bất đẳng thức, cực trị tổ hợp hay một số bài toán chứng minh sự tồn tại.

Để sử dụng phương pháp đếm bằng hai cách, đòi hỏi học sinh phải biết và vận dụng tốt các phép đếm cơ bản. Bài viết này được sử dụng để giảng dạy cho học sinh lớp 10 chuyên Toán, các em mới bước đầu làm quen với các bài toán tổ hợp nói chung và các bài toán đếm nói riêng nên ví dụ được nêu ra có độ khó không cao giúp các em làm quen với phương pháp này. Vì thời gian quá gấp rút nên không tránh khỏi sai sót, bạn đọc có thắc mắc xin liên hệ địa chỉ nguyentangvu@gmail.com,cảm ơn.

1. Chứng minh các đẳng thức tổ hợp
Ví dụ 1. Cho các số nguyên dương $ n $ và $ k $ với $ 0 < k \leq n $. Chứng minh các đẳng thức tổ hợp sau:

a) $ C_n^k=C^k_{n-1}+C^{k-1}_{n-1} $

b) $ \sum_{k \geq 0}C^{2k}_n=2^{n-1} $

Giải

a) Dễ thấy vế trái của đẳng thức là số cách chọn $ k $ phần tử từ  $ n $  phần

tử. Để chọn $ k $ phần tử từ $ n $ phần tử ta có thể làm như sau: Xét phần tử

$ a $, nếu $ a $ được chọn thì ta cần chọn thêm $ k−1 $ phần tử từ $ n−1 $

phần tử còn lại ta có $ C^{k−1}_{ n−1} $ cách. Nếu $ a $ không được chọn,

ta chọn $ k $ phần tử từ $ n−1 $ phần tử còn lại, ta có $ C^k_ {n−1} $. Do

đó số cách chọn trong hai trường hợp là $C^k_{n-1}+C^{k-1}_{n-1} $. Từ

đó ta có điều cần chứng minh.

b) Ta xét bài toán “đếm số cách chọn một số chẵn phần tử từ $ n $ phần tử”.Ta có thể đếm theo cách sau:

Cách 1: Ta có số cách chọn $ 2k $ phần tử từ $ n $ phần tử là $ C^{2k}_n $ . Khi

đó $ \sum_{k \geq 0}C^{2k}_n $ lần tổng số cách chọn một số chẵn phần tử từ

$ n $ phần tử.

Cách 2: Xét một phần tử $ a $, thì có hai khả năng $ a $ được chọn hoặc $ a $

không được chọn, ta có 2 trường hợp. Khi đó với $ n−1 $ phần tử đầu tiên, thì

số trường hợp là $ 2^{n−1} $. Tới phần tử thứ $ n $, nếu ta đã chọn được một

số chẵn phần tử thì ta không chọn, còn nếu ta đã chọn được một số lẻ phần

tử thì phần tử này sẽ được chọn, do đó số cách chọn là $ 2^{n−1} $.

Ví dụ 2. Cho các số nguyên dương $ n $ và $ k $ với $ 0 \leq k \leq n $. Chứng minh rằng:

a) $ kC^k_n=nC^{k-1}_{n-1} $

b) $ \sum_{k=0}^{n}kC^k_n=n2^{n-1} $

Giải
a) Xét bài toán “Một đội văn nghệ có n thành viên, có bao nhiêu cách chọn

$k$ người thể hiện một tiết mục hát tốp ca trong đó có một bạn hát sô lô”.

Cách 1: Chọn đội văn nghệ gồm $ k $ người từ $ n $ ta có số cách là $ C^k_n $,

từ $ k $ người này ta chọn một người hát sô lô có $ k $ cách. Khi đó số cách

chọn là $ kC^k_n $.(1)

Cách 2: Chọn người hát sô lô trước, có $ n $ cách, sau đó chọn $ k−1 $ người từ

$ n−1 $ người còn lại có $ C^{k−1}_{n−1} $ cách.

Vậy số cách chọn là $ nC^{k−1}_{n−1} $. (2)

Từ (1) và (2) ta có đẳng thức $ kC^k_n = nC^{k−1}_{n−1}. $

b) Xét bài toán “Từ $ n $ thành viên của đội văn nghệ, có bao nhiêu cách lập một nhóm hát trong đó có một nhóm trưởng?”. Làm tương tự như câu trên ta sẽ có đẳng thức cần chứng minh.

Ví dụ 3. Cho các số nguyên dương $ n $ và $ k $ với $ 0 \leq k \leq n $. Chứng minh rằng:
a) $ \sum_{m=k}^{n}C^k_m=C^{k+1}_{n+1} $

b) $ \sum_{m=k}^{n-k}C^k_mC^k_{n-m}=C^{2k+1}_{n+1} $

với $ 0 \leq k \leq\dfrac{n}{2} $.

Giải

a) Xét tập $ X = {1,2,…,n + 1} $. Khi đó ta đếm số tập con có $ k + 1 $ phần tử của $ X $.

Cách 1: Rõ ràng số tập con là $ C^{k+1}_{ n+1} $.

Cách 2: Ta chọn tập con sao cho phần tử lớn nhất là $ m $. Khi đó số tập con

có phần tử lớn nhất $ m $ là $ C^k_m $. Vì $ k \leq m \leq n $ nên ta có số tập

con là $ C^k_k + C^k_{k+1} + … + C^k_n $. Từ đó suy ra đẳng thức cần chứng

minh.

b) Xét bài toán “Đếm số tập con có $ 2k+1 $ phần tử của $ X $”.

Cách 1: Số tập con là $ C^{2k+1}_n $.

Cách 2: Ta xét phần tử thứ $ k + 1 $, giả sử đó là $ m $, khi đó ta chọn $ k $

phần tử nhỏ hơn $ m $ và $ k $ phần tử lớn hơn $ m $, số cách chọn là

$ C^k_mC^k_{n−m} $, vì $ k \leq m \leq n−k $ nên ta có số cách chọn là

$ \sum_{m=k}^{n-k} C^k_mC^k_{n-m}$.

Từ đó ta có đẳng thức cần chứng minh.

Bài tập

Bài 1 Cho $ 0 \leq k \leq m \leq n. $ Chứng minh các đẳng thức sau:

a) $ C^k_mC^m_n=C^k_nC^{m-k}_{n-k} $

b) $ \sum_{k \geq 0}k(C^k_n)^2=nC^{n-1}_{2n-1} $

c) $ \sum_{k \geq 0}C^k_nC^{m-k}_{n-k}=2^mC^m_n $

Bài 2. Chứng minh các đẳng thức sau:

a) $\sum_{i=0}^{k} C^i_n C^{k-i}_{n-i} = 2^kC^k_n$

b) $ kC^k_m C^0_p+(k-1)C^{k-1}_m C^1_p+…+C^1_mC^{k-1}_p$

$=\dfrac{m}{m+p}.k.C^k_{m+p} $

Ví dụ 4. Trong một hội nghị, mỗi thành viên tham gia đúng 3 cuộc họp và mỗi cuộc họp thì có đúng 6 thành viên tham gia. Chứng minh rằng số cuộc họp thì bằng nửa số thành viên tham gia hội nghị.

Giải
Gọi số thành viên là $ n $, số cuộc hộp là $ m $. Khi đó mỗi cuộc họp có 6 thành viên tham gia, nên tổng số lượt thành viên tham gia $ m $ cuộc họp là $ 6m $ (có lặp lại). Tương tự mỗi thành viên tham gia 3 cuộc họp mà có $ n $ thành viên nên số lượt thành viên tham gia là $ 3n $. Do đó $ 3n = 6m $ hay $ n = 2m $.
Trong bài toán trên ta có thể làm như sau: giả sử có $ m $ cuộc họp là $ 1,2,…,m $ và $ n $ thành viên là $ 1,2,3,…,n $. Xét bảng vuông $ m \times n $ gồm $ m $ dòng và $ n $ cột trên đó ghi các số dòng thứ $ i $ cột $ j $ là $ aij $ thỏa $ aij = 1 $ nếu người $ j $ tham gia cuộc họp thứ $ i $ và $ a{ij} = 0 $ trong trường hợp ngược lại. Ta được bảng sau:


Dựa vào trên, ta thấy mỗi dòng có 6 số 1 và mỗi cột có 3 số 1. Khi đó ta có $ 6m = 3n $ hay $ n = 2m $.
Bảng trên được gọi là một ma trận nhị phân, dùng để biểu diễn các mối quan hệ hai ngôi như phần tử thuộc tập hợp, quen nhau, đồ thị… và là mô hình biểu diễn rất hữu dụng trong các bài toán tổ hợp. Trong mỗi bảng nhị phân trên, nếu gọi $ r_i $ là số số 1 ở dòng thứ $ i $ và $ c_j $ là số số 1 ở cột thứ $ j $, ta có :
$ \sum_{i=1}^{m}r_i=\sum_{j=1}^{n}cj $

Ví dụ 5 (HK 1994) Trong một trường học có $ m $ giáo viên và $ n $ học sinh thỏa điều kiện sau:
i) Mỗi giáo viên dạy đúng p học sinh.
ii) Với hai học sinh phân biệt thì có đúng $ q $ giáo viên dạy họ.
Chứng minh rằng $ \dfrac{m}{q}=\dfrac{n(n-1)}{p(p-1)} $

Giải
Lập bảng gồm $ m $ dòng và $ n $ cột trong đó $ aij = 1 $ nếu giáo viên $ i $ dạy học sinh $ j $, và bằng $ 0 $ nếu ngược lại. Khi đó từ (i) thì mỗi dòng có đúng $ p $ số $ 1 $. Ta đếm các cặp số $ (1;1) $ trên cùng một dòng. Nếu đếm theo dòng thì mỗi dòng có $ C^2_p $ cặp, có $ m $ dòng nên số cặp là $ mC^2_p $. (1)
Nếu đếm theo cột, do điều kiện (ii) nên với hai cột bất kì thì có đúng $ q $ cặp. Do đó số cặp là $ qC^2_n $ (2). Từ (1) và (2) ta có $ mC^2_p=qC^2_n $ hay $ \dfrac{m}{q} =\dfrac{n(n-1)}{p(p-1)}$.

Trên đây là một kĩ thuật đếm theo cặp $ (1;1) $ cùng một dòng hoặc cùng một cột. Ta có mệnh đề sau:

Định lý 1. Nếu trong một bảng nhị phân $ m \times n, $ mỗi dòng có $ k $ số 1, hai cột bất kỳ có đúng $ p $ cặp $ (1;1) $ cùng một dòng.

Khi đó ta có $ pC^2_n=kC^2_m. $

Bài tập
Bài 1. Cho tập $ X = {1,2,…,8} $ và các tập $ A1,A2,…,A6 $ là các tập con của $ X $ sao cho mỗi tập $ Ai $ có $ 4 $ phần tử và mỗi phần tử của $ S $ thuộc $ m $ tập $ Ai $. Tìm $ m $.

Bài 2. Trong một vòng thi toán chung kết tại trường PNTK, các thí sinh phải giải 9 bài toán. Biết rằng mỗi thí sinh giải được đúng 6 bài, và với hai thí sinh bất kì thì giải đúng chung 3 bài. Tìm số thí sinh dự thi.
Bài 3. Gọi $ p(n,k) $ là số hoán vị của $ {1,2,…,n} $ có $ k $ điểm bất động. Chứng minh rằng:
$ \sum_{k=1}^{n}kp(n,k)=n! $

2. Chứng minh các bài toán bất đẳng thức và cực trị tổ hợp

Ví dụ 6. (Iran 2011) Cho $ n $ điểm trên mặt phẳng sao cho không có 3 điểm nào thẳng hàng. Chứng minh rằng số tam giác có diện tích bằng 1 có các đỉnh thuộc $ n $ điểm trên không vượt quá $ \dfrac{2}{3}(n^2-n) $.

Giải
Bài toán này ta đi tính số cặp (cạnh;tam giác). Với đoạn thẳng $ AB $, khi đó nếu điểm $ C $ thỏa $ S_{ABC} = 1 $ thì khoảng cách từ $ C $ đến $ AB $ bằng $ \dfrac{2}{AB} $ , vì không có 3 điểm nào thẳng hàng nên chỉ có nhiều nhất 4 điểm thỏa. Vậy với 1 đoạn ta sẽ có nhiều nhất 4 tam giác có diện tích 1 nhận đoạn thẳng đó làm đỉnh. Suy ra tổng số cặp nhiều nhất là $ 4C^2_n $.
Mặt khác nếu gọi số tam giác là $ m $ thì tổng số cặp là $ 3m $.
Từ đó ta có: $ 3m \leq 4C^2_n $ hay $ m \leq \dfrac{2}{3}(n^2-n) $.

Ví dụ 7.(USA TST 2005) Cho $ n > 1 $. Với số nguyên dương $ m $. Đặt $ X_m = {1,2,…,mn} $. Xét họ $ T $ gồm $ 2n $ tập hợp thỏa các điều kiện sau:
i) Mỗi phần tử của $ T$ là một tập con có $ m $ phần tử của $ X_m. $
ii) Mỗi cặp thuộc $ T $ có nhiều nhất một phần tử chung.
iii) Mỗi phần tử thuộc $ X_m $ thuộc đúng hai tập của $ T. $
Tìm giá trị lớn nhất của $ m $ theo $ n. $

Giải

Xét bảng vuông sao cho gồm $ 2n $ dòng và $ mn $ cột sao cho $ a_{ij} =1$ nếu số $ j $ thuộc $ a_i $ và bằng $ 0 $ trong trường hợp ngược lại.
Ta xét bài toán đếm số cặp $ (1;1) $ cùng một cột. Do (i) nên ta có số cặp nhiều nhất là $ C^2_{2n} $.
Do (ii) nên ta có số cặp là $ mn $.
Do đó $ mn \geq C^2_ {2n} $, suy ra $ m \geq 2n−1 $. Nếu $ m = 2n−1 $, ta xét mô hình sau. Cho $ 2n $ đường thẳng không có 3 đường nào đồng quy và không có hai đường nào song song. Khi Xm là tập các giao điểm và $ T $ là họ gồm các điểm thuộc một đường thẳng. Rõ ràng đây là mô hình thỏa đề bài. Bảng sau cho ví dụ $ n=2, m=3 $.

Ví dụ 8. (IMO 1998, P2) Trong một cuộc thi có $ a $ thí sinh và $ b $ giám khảo, với $ b $ là số lẻ lớn hơn 3. Mội giám khảo có thể đánh giá thí sinh rớt hay đậu.Giả sử với hai giám khảo bất kì thì quyết định giống nhau nhiều nhất là $ k $ thí sinh. Chứng minh rằng $ \dfrac{k}{a} \geq \dfrac{b-1}{2b} $
Giải
Cũng như ví dụ trên, ta thấy việc biểu diễn các mối quan hệ bằng bảng nhị phân rất thuận lợi trong việc trình bày lời giải. Trong bài này ta cũng có thể lập bảng $ b \times a $ theo quy tắc sau: dòng i cột j bằng 1 nếu giám khảo i cho thí sinh j đậu. Ta sẽ đếm số cặp $ (0;0) $ và $ (1;1) $ cùng một cột bằng hai cách.
Cách 1 ta đếm theo dòng: Vì với hai vị giáo bất kì có nhiều nhất $ k $ kết luận giống nhau nên với hai dòng bất kì có $ k $ cặp, do đó số cặp nhiều nhất là $ kC^2_b $.
Cách 2 ta đếm theo cột: Trong mỗi cột số cặp là $ C^2_m+C^2_n $ cặp, trong đó $ m $ là số các số $ 0 $ và $ n $ là số các số $ 1, $ ta có $ m+n=b=2t+1, $ suy ra $ n=2t+1-m. $
Khi đó $ C^2_m+C^2_n=\dfrac{m(m-1)+(21-m)(2t-m-1)}{2}=\dfrac{(2t-m)^2+m^2}{2} \geq t^2=\dfrac{(b-1)^2}{4}. $
Từ đó ta có $ kC^2_b \geq \dfrac{a(b-1)^2}{4} $, suy ra $ \dfrac{k}{a} \geq \dfrac{b-1}{2b.} $

Ví dụ 9. Cho $ n $ điểm trong mặt phẳng. Chứng minh rằng số cặp điểm có

khoảng cách bằng 1 không quá $ \dfrac{n}{4}+\dfrac{\sqrt{2n^3}}{2}. $

Giải

Gọi $ d_i $ là số đoạn thẳng có độ dài 1 mà có đỉnh là $ A_i $. Đặt khi

đó số cặp điểm là $ k = \dfrac{1}{2} (d_1 + d_2 + … + d_n) $. Ta đếm số cặp

$ (A,B) $ mà khoảng cách từ $ A,B $ đến $ A_i $ bằng 1. Số cặp là $ C^2 _{di} $,

suy ra tổng số cặp là $ \sum_{i=1}^{n}C^2_{d_i} $. Ta biết rằng hai điểm $ C,D $

thì có chung nhiều nhất một cặp $ (A,B) $ nên số cặp không vượt quá

$ 2C^2_n $. Do đó: $ \sum_{i=1}^{n}C^2_{d_i} \leq n(n-1) $

hay $ \dfrac{2k(2k-n)}{2n} \leq n(n-1) \Leftrightarrow 2k^2-nk-n^2(n-1) \leq 0 $

Do đó $ k \leq \dfrac{n}{4}+\dfrac{\sqrt{2n^3}}{2} $

3 Các bài toán tồn tại
Ví dụ 10. Cho 133 số nguyên dương, có ít nhất 799 cặp số là nguyên tố cùng nhau. Chứng minh rằng tồn tại 4 số nguyên dương phân biệt $ a,b,c,d $ sao cho $ a $ và $ b; b $ và $ c, c $ và $ d; d $ và $ a $ nguyên tố cùng nhau.

Giải

Mỗi số được đại diện bởi một điểm, hai số nào nguyên tố cùng nhau thì hai điểm tương ứng được nối nhau bởi một đoạn. Ta cần chứng minh có 4 đoạn $ AB,BC,CD,DA $. Ta cần chứng minh rằng có hai điểm $ B $ và $ D $ cùng nối với hai điểm $ A $ và $ C $.
Gọi $ d_i $ là số cạnh có đỉnh là $ A_i $. Khi đó ta có

$ d_1 + d_2 + … + d_{133} = 2 \times 799 $. Nếu hai đỉnh $ Y, Z $ cùng nối với đỉnh $ X $ thì ta sẽ xem $ (Y;Z) $ là một cặp. Ta sẽ tính số cặp này. Rõ ràng, tổng số cặp là
$ \sum_{i=1}^{133} C^2_{d_i}$

$=\dfrac{1}{2}\left(\sum_{i=1}^{133}d^2_i-\sum_{i=1}^{133}d_i \right) $

Ta có

$ \sum_{i=1}^{133}d^2_i \geq \dfrac{1}{133} \left(\sum_{i=1}^{133}d_i \right)^2 $

Do đó

$ \sum_{i=1}^{133}C^2_{d_i} \geq \dfrac{1}{2}(\dfrac{1}{133}(\sum_{i=1}^{133}d_i)^2)$

$-\sum_{i=1}^{133}d_i ]>C^2_{133} $
Nhưng $ 133 $ điểm thì có $ C^2_{133} $ cặp, nên sẽ có một cặp nào đó được tính hai lần, tức là tồn tại cặp $ (A,C) $ cùng được nối với $ B$ và $ D $. Tức là ta có 4 đoạn $ AB, BC,CD,DA. $

Ví dụ 11.  Cho tập $ X $ có $ n $ phần tử, gọi $ A_1,A_2,…,A_m $ là một họ các tập con của $ X $, sao cho $ |Ai| = 3 $ và $ |A_i \cap A_j| \leq 1 $ với $ i \neq j $. Chứng minh rằng tồn tại một tập con $ A $ của $ X $ có ít nhất $ [\sqrt{2n}] $ phần tử và không chứa bất kì tập $ A_i $ nào.

Giải
 

Ta xét tập tất cả các tập con của $ X $ mà không chứa bất kỳ tập $ A_i $ nào, khi đó dễ thấy tập này là khác rỗng (xét tập có 2 phần tử là thỏa) và hữu hạn, nên tồn tại một tập $ M $ có nhiều phần tử nhất. Đặt $ |M|=k $. Khi đó, do $ M $ có số phần tử lớn nhất nên mọi tập có số phần tử lớn hơn $ M $ đều chứa một tập $ A_i. $ Xét tập $ M’=X \setminus M= \{a_1,a_2,…,a_{n-k}\} $. Khi đó $ M \cup \{a_i\} $ có $ k+1 $ phần tử, nên theo cách xác định $ M $ thì sẽ tồn tại $ A_i \subset M’,$ do $ A_i \nsubseteq M $ nên $ A_i=\{a_i,x,y\} $ trong đó $ x,y \in M. $
Hơn nữa hai tập giao nhau có không quá một phần tử nên với mỗi $ a_i $ có nhiều nhất một cặp $ (x,y) \in A_i$. Ta đếm số cặp $ (x,y) $ theo hai cách:\\
Số cặp $ (x,y) \in X $ là $ C_k^2. $
Vì $ i=1,2,…,n-k $ nên có $ n-k $ cặp. Vậy ta có:
$ n-k \leq C^2_k \Leftrightarrow k^2+k \geq 2n $
Mà $ k \leq \sqrt{k^2+k} \leq k+1, $ suy ra $ k \geq [\sqrt{2n}] $. Ta có điều cần chứng minh.

Bài tập rèn luyện
Bài 1.  Cho 7 tập $ A1,A2,…,A7 $ là các tập con của $ X = {1,2,3,4,5,6,7} $, sao cho mội cặp phần tử thuộc $ X $ thuộc đúng một tập con, và $ |Ai|\geq 3 $ với mọi $ i $. Chứng minh rằng $ |A_i \cap Aj| = 1 $ với mọi $ i,j. $

Bài 2. Cho 16 bạn học sinh làm một bài kiểm tra trắc nghiệm, trong đó mỗi câu hỏi có 4 lựa chọn. Sau bài kiểm tra, ta thấy rằng với hai học sinh bất kì có nhiều nhất một câu trả lời giống nhau. Hỏi bài kiểm tra có nhiều nhất bao nhiêu câu hỏi?

Bài 3. Một hội nghị có n thành viên tham gia, hội nghị đã tổ chứng $ n + 1 $ cuộc họp, trong đó mỗi cuộc họp có đúng 3 người và không có cuộc họp nào có thành viên giống nhau. Chứng minh rằng có hai cuộc họp mà có chung đúng một thành viên.

Bài 4.  (China 1996) Trong một hội nghị có 8 người tham gia, hội nghị tổ chức $ m $ cuộc họp, mỗi cuộc họp có đúng 4 người tham gia. Hơn nữa hai người bất kì thì cùng tham gia một số cuộc họp như nhau. Tìm giá trị nhỏ nhất của $ m $.
Bài 5.  Cho $ A1,A2,…,Ak $ là các tập con của $ S = {1,2,…,10} $ sao cho:
i) $ |A_i| = 5,i = 1,2,…,k. $
ii) $ |A_i \cap A_j| \leq 2, 1 \leq i < j \leq k. $ Tìm giá trị lớn nhất của $ k $.

Bài 6. (IMO 2001) Có 21 bạn nam và 21 bạn nữ tham dự một kì thi học sinh giỏi toán. Biết rằng:
a) Mỗi bạn giải được nhiều nhất sáu bài.
b) Mỗi cặp một nam và một nữ thì có ít nhất một bài toán được giải bởi hai người đó.
Chứng minh rằng có môt bài toán mà giải được bởi ít nhất 3 nam và 3 nữ.

Bài 7.  (USAMO 2001) Có 8 hộp, mỗi hộp chứa 6 viên bi. Mỗi viên bi được tô màu sao cho:
i) Mội hộp chứa các viên bi khác màu.
ii) Không có hai màu nào cùng xuất hiện nhiều hơn trong một hộp.
Tìm số màu ít nhất cần dùng.

Bài 8.  (IMO 1989) Cho $ n $ và $ k $ là các số nguyên dương và $ S $ là tập $ n $ điểm trong mặt phẳng sao cho:
i) Không có 3 điểm nào thẳng hàng,
ii) Với điểm $ P $ bất kì thuộc $ S $ thì có ít nhất $ k $ điểm của $ S $ cách đều $ P $.
Chứng minh rằng: $ k<\dfrac{1}{2}+\sqrt{2n} $

Bài 9. (IMO 2005) Trong một cuộc thi toán trong đó đề thi có 6 bài. Mỗi một cặp bài toán được giải bởi nhiều hơn $ \dfrac{2}{5} $ số thí sinh. Không có ai giải được 6 bài. Chứng minh rằng có ít nhất 2 thí sinh giải được đúng 5 bài.

Bài 10. Trong một hội nghị có 35 người tham gia. Biết rằng có 111 cặp đôi một quen nhau. Chứng minh rằng có thể chọn ra 4 thành viên xếp ngồi vào một bàn tròn sao cho hai người ngồi gần nhau thì quen nhau.

 

Biến đổi góc – Phần 2

Ví dụ 5. (Đề thi HSG Quốc Gia Việt Nam năm 2014) Cho tam giác nhọn $ABC$ nội tiếp đường tròn $(O)$ với $AB < AC$. Gọi $I$ là trung điểm cung $BC$ không chứa $A$. Trên $AC$ lấy điểm $K$ khác $C$ sao cho $IK = IC$. Đường thẳng $BK$ cắt $(O)$ tại $D$ $(D \neq B)$ và cắt đường thẳng $AI$ tại $E$. Đường thẳng $DI$ cắt đường thẳng $AC$ tại $F$.

a. Chứng minh rằng $EF = \dfrac{BC}{2}$.
b. Trên $DI$ lấy điểm $M$ sao cho $CM$ song song với $AD$. Đường thẳng $KM$ cắt đường thẳng $BC$ tại $N$. Đường tròn ngoại tiếp tam giác $BKN$ cắt $(O)$ tại $P$ $(P\neq B)$. Chứng minh rằng đường thẳng $PK$ đi qua trung điểm đoạn thẳng $AD$.

Giải

a.

  • Chứng minh $\angle AKI = \angle ABI$ (cùng bù $\angle ACI$).
  • Tam giác $ABI, AKI$ bằng nhau, suy ra $E$ là trung điểm của $BK$.
  • Chứng minh $F$ là trung điểm $CK$.

b.

  • Tam giác $AID$ có $DE, AF$ là đường cao cắt nhau tại $K$ nên $K$ là trực tâm, suy ra $IK \bot AD$, do đó $CM \bot IK$. Suy ra $M$ là trực tâm tam giác $IKC$.
  • Khi đó $AC$ là tiếp tuyến của $(BKN)$.
  • $\angle CKP = \angle KBP = \angle DIP$, suy ra $KFPI$ nội tiếp, do đó $\angle IPK = 90^\circ $, suy ra $IJ$ là đường kính.
  • Từ đó chứng minh $JAKD$ là hình bình hành.

 

Ví dụ 6.  (Trần Quang Hùng) Cho tam giác $ABC$ nhọn, nội tiếp đường tròn tâm $O$. Các đường cao $AD, BE, CF$ cắt nhau tại $H$, $AD$ cắt $(O)$ tại $K$. $KF$ cắt $(O)$ tại $L$.
a. Chứng minh $CL$ đi qua trung điểm của $EF$.
b. Đường thẳng qua $A$ song song với $DE$ cắt $CL$ tại $N$. Chứng minh $\angle OFN = 90^\circ$.

Giải

a.

  •  Gọi $P$ là giao điểm của $CL$ và $DE$, $HP$ cắt $AC$ tại $D$.
  • Ta có $\angle CH \cdot CF = \angle CA \cdot CE = CP \cdot CL$ nên $LFHP$ nội tiếp.
  • Suy ra $\angle CHP = \angle CLF = \angle CAD = \angle CFE$, do đó $HP \parallel FE$.
  • Ta có $EH$ là phân giác $\angle DEF$, suy ra $\angle PHE = \angle HEF = \angle HEP$, suy ra $PE = PH$.
  • Tam giác $HES$ vuông, suy ra $P$ là trung điểm $HS$. Từ đó ta có $M$ là trung điểm của $EF$.

b.

  • Ta chứng minh $\triangle FAN \backsim \triangle FOC$ đồng dạng. Vì có $\angle FCO = \angle FAN$ nên ta chỉ cần chứng minh $\dfrac{NA}{OC} = \dfrac{AF}{CF}$. \hfill (1)
  • Trong đẳng thức trên chỉ có $AN$ có vẻ là chưa liên quan gì, nên ta tính $AN$ trước. Ta có $\dfrac{AN}{PE} = \dfrac{AC}{CE}$, suy ra $AN=\dfrac{AC \cdot PE}{CE}$.
  • Ta có $PE = \dfrac{1}{2} HS = \dfrac{CH \cdot EF}{2CF}$.
  • Suy ra $\dfrac{AN}{OC} = \dfrac{CA \cdot EF \cdot CH}{CE \cdot CF \cdot 2OC}$, ta cần chứng minh $\dfrac{CA \cdot EF \cdot CH}{CE \cdot CF \cdot 2OC} = \dfrac{AF}{CF} $
  • $\Leftrightarrow \dfrac{AF}{EF} = \dfrac{CA}{CE}\cdot \dfrac{CH}{2R}$
  • $\Leftrightarrow \dfrac{AC}{AB} = \dfrac{CA}{CE}\cdot \dfrac{CH}{2R}$
  • $\Leftrightarrow \dfrac{CE}{CH} = \dfrac{AB}{2R}$ (Đúng).

Ví dụ 7. Cho tam giác $ABC$ vuông tại $A$, dường cao $AD$, trên đoạn $AD$ lấy điểm $E$, trên tia $BE, CE$ lấy các điểm $F, L$ sao cho $CL = CA, BF = BA$. $BF, CL$ cắt nhau tại $K$. Chứng minh rằng tam giác $KFL$ cân.

Giải

  • Gọi $M, N$ là giao điểm của $BE, CE$ với $(ABC)$.
  • Khi đó $CM, BN, AD$ đồng quy tại $H$.
  • Ta có $BN\cdot BH = BD\cdot BC = BA^2 = BF^2$. Suy ra $BF \bot AF$. Tương tự thì $CL \bot AL$.
  • $AF^2 = AN\cdot AB = AM\cdot AC = AL^2$. Suy ra $AF = AL$. Từ đó ta có $KF = KL$.

Ví dụ 8. Cho tam giác $ABC$ nội tiếp đường tròn $w$. Gọi $I$ là tâm đường tròn nội tiếp của tam giác $ABC$. Các đường thẳng $AI, BI, CI$ cắt $w$ lần lượt tại $A’,B’, C’$. $M$ là một điểm trên cạnh $AB$. Đường thẳng qua $M$ và song song với $AI$ cắt đường thẳng qua $B$ vuông góc với $BI$ tại điểm $A_1$; đường thẳng qua $M$ song song với $BI$ và cắt đường thẳng qua $A$ vuông góc với $AI$ tại điểm $B_1$. Chứng minh rằng $A’A_1, B’B_1$ và $C’M$ đồng quy.

Giải

  • Gọi $T$ là giao điểm của $B’B_1$ và $(O)$. Ta có $\angle MB_1T = \angle BB’T = \angle MAT$, suy ra tứ giác $AMTB_1$ nội tiếp, kéo theo $\angle AB_1M = \angle ATM $ . \hfill (1)
  • Ta chứng minh được $B’C’ \bot AA’$, suy ra $AB_1 \parallel B’C’$, từ đó ta có $\angle AB_1M = \angle C’B’lB$. \hfill (2)
  •  Từ (1) và (2), suy ra $T, M$ và $C’$ thẳng hàng. Chứng minh tương tự thì giao điểm của $A’A_1$ và $(O)$ cũng thuộc $C’M$. Từ đó ta có điều cần chứng minh.

Bài tập rèn luyện.

Bài 1. Cho hai điểm $P, Q$ thuộc miền trong của tam giác $ABC$ sao cho $$\angle ACP = \angle BCQ, \angle CAP = \angle BAQ$$ Gọi $D, E, F$ là hình chiếu vuông góc của $P$ trên các đường thẳng $BC, AC, AB$. Chứng minh rằng nếu $\angle DEF = 90^\circ$ thì $Q$ là trực tâm của tam giác $BDF$.

Bài 2. Cho tam giác nhọn $ABC$ nội tiếp đường tròn $(O)$. Các đường cao $AD$ và $BD$. Gọi $M$ là trung điểm $AB$, phân giá trong góc $\angle BCA$ cắt $DE$ tại $P$ và cắt $(O)$ tại $Q$. Gọi $C’$ là điểm đối xứng của $C$ qua $AB$. Tính $\angle C$ biết rằng 4 điểm $M, P, Q$ và $C’$ cùng thuộc một đường tròn.

Bài 3. Cho đường tròn $(O)$ và điểm $P$ nằm ngoài đường tròn. Vẽ các tiếp tuyến $PA, PB$ đến $(O)$ với $A, B$ là các tiếp điểm. $C$ là điểm trên cung nhỏ $AB$, tiếp tuyến tại $C$ cắt $PA, PB$ và $PO$ lần lượt tại $D, E, F$. Chứng minh rằng đường tròn ngoại tiếp các tam giác $PAB, PDE$ và $PCF$ cùng đi qua một điểm khác $P$.

Bài 4. (Đề thi chon đội dự tuyển PTNK năm 2009) Cho tam giác $ABC$ nhọn. Trên các tia đối của các tia $BC, CA, AB$ lấy các điểm $A_1, B_1, C_1$ sao cho tam giác $A_1B_1C_1$ đồng dạng với tam giác $ABC$. Chứng minh rằng trực tâm tam giác $A_1B_1C_1$ cũng là tâm đường tròn ngoại tiếp tam giác $ABC$.

Bài 5. (Đề thi HSG Toán Quốc Tế năm 2009) Cho tam giác $ABC$ cân tại $A$. Phân giác trong góc $A$ và $B$ cắt $BC$ và $AC$ lần lượt tại $D$ và $E$. Gọi $K$ là tâm đường tròn nội tiếp tam giác $ACD$. Cho $\angle BEK = 45^o$. Tìm tất cả các giá trị của $\angle BAC$.

Bài 6. (Đề thi toán Quốc tế năm 2017) Cho $R,S$ là hai điểm phân biệt trên đường tròn $\Omega$ sao cho $RS$ không phải đường kính. Gọi $d$ là tiếp tuyến của $\Omega$ tại $R$. Lấy điểm $T$ sao cho $S$ là trung điểm của đoạn thẳng $RT$. Lấy điểm $J$ trên cung nhỏ $RS$ của $\Omega$ sao cho $(JST)$ cắt $d$ tại hai điểm phân biệt. Gọi $A$ là giao điểm gần $R$ nhất của $d$ và $(JST)$. $AJ$ cắt lại $\Omega$ tại $K$. Chứng minh $KT$ tiếp xúc với $(JST)$.

Bài 7. (Đề thi HSG Bulgari năm 2016) Cho tam giác $ABC$ cân tại $C$, trên tia đối của tia $CA$ lấy điểm $D$ sao cho $AC > CD$. Phân giác $\angle BCD$ cắt $BD$ tại $N$. $M$ là trung điểm $BD$, tiếp tuyến tại $M$ của $(AMD)$ cắt $BC$ tại $P$. Chứng minh rằng 4 điểm $A, P, M, N$ cùng thuộc một đường tròn.

Biến đổi góc – Phần 1

Một trong những kĩ năng làm toán hình học đó là chứng minh các góc bằng nhau hay so sánh các góc, để dẫn tới các tam giác bằng nhau hay tam giác đồng dạng. Do đó kĩ năng biến đổi góc chiếm vị trí quan trọng trong việc chứng minh các tính chất hình học, vì thế chương đầu tiên của sách này tôi đưa ra một số bài toán liên quan đến việc tính toán, so sánh các góc, từ đó giải quyết được yêu cầu bài toán.

Việc tính toán các góc, tôi ưu tiên cho góc hình học mà không sử dụng góc định hướng. Việc sử dụng góc hình học phụ thuộc và hình vẽ nên lời giải nhiều khi không mang tính tổng quát, tuy vậy đối với các em mới từ lớp 9 lên thì cách trình bày này dễ tiếp thu hơn, và thực sự đối với số đông cũng vậy. Việc vẽ hình đó cũng là kĩ năng của người làm hình học, chú ý các trường hợp đề bài nêu ra để vẽ hình chính xác yêu cầu, từ đó có lời giải phù hợp. Chương trình vẽ hình trong sách là geogebra đã rất phổ biến với cộng đồng làm toán sơ cấp, tôi sẽ dùng chương trình này hỗ trợ làm tài liệu này. Có một điều khuyên cho các em học sinh là hãy vẽ bằng tay và dùng compa thước, không nên dùng phần mềm hỗ trợ để vẽ, vì khi thi cử thì không dùng máy để vẽ hay phát hiện tính chất.

Kiến thức chính của chương này là các kiến thức liên quan đến góc và đường tròn, tam giác đồng dạng, tứ giác nội tiếp đã học trong chương trình THCS. Các bài toán cũng chỉ sử dụng kiến thức của trung học cơ sở để giải.

Ví dụ 1. (Định lý Migel) Cho tam giác $ABC$. Các điểm $D, E, F$ lần lượt thuộc các đường thẳng $BC, AC$ và $AB$.
a. Chứng minh rằng đường tròn ngoại tiếp các tam giác $AEF, BDF, CDE$ cùng đi qua một điểm. Điểm này được gọi là điểm Migel.
b. Chứng minh điểm Migel thuộc đường tròn ngoại tiếp của tam giác $ABC$ khi và chỉ khi $D, E, F$ thẳng hàng.
c. Khi $D, E, F$ thẳng hàng. Chứng minh rằng tâm đường tròn ngoại tiếp của các tam giác $AEF, BDF, CDE$ và điểm $P$ cùng thuộc một đường tròn.

Giải

a. Gọi $P$ là giao điểm của $(AEF)$ và $(BDF)$. Ta có $\angle PDC = \angle BFP = \angle AEP$. Suy ra $CDPE$ nội tiếp, hay $P \in (CDE)$. \\Vậy $(AEF), (BDF)$ và $(CDE)$ cùng đi qua một điểm.

b. Khi $D, E, F$ thẳng hàng.
Ta có $\angle DPB = \angle PFD = \angle PAE = \angle PAC$. Suy ra $P \in (ABC)$.
Ngược lại nếu $P \in (ABC)$ ta có $\angle PFD = \angle PBD = \angle PAE = 180^\circ – \angle PFE$. Suy ra $D, E, F$ thẳng hàng.

c.  

  • Gọi $O, O_a, O_b, O_c$ lần lượt là tâm đường tròn ngoại tiếp các tam giác $ABC, AEF, BDF, CDE$.
  • Gọi $H = O_bO_c \cap PD, K = OO_a \cap PB, L = OO_c \cap PC$.
    Ta có $O_bOc$ là trung trực của $PD$ nên $H$ là trung điểm của $PD$ và $\angle PH \bot O_bO_c$; tương tự với $K, L$.
  • $H, K, L$ là hình chiếu của $P$ trên các đường thẳng chứa các cạnh của tam giác $OO_bO_c$, dễ thấy $H, K, L$ thẳng hàng nên 4 điểm $P, O, O_b, O_c$ cùng thuộc đường tròn. (Định lý đảo của đường thẳng Simson).
  • Tương tự cho $P, O, O_a, O_c$ cũng cùng thuộc một đường tròn. Vậy 5 điểm $P, O, O_a, O_b, O_c$ cùng thuộc một đường tròn.

 

Ví dụ 2. (Đề đề nghị IMO 2002) Cho đường tròn $w$, $B$ là một điểm $w$. Trên tiếp tuyến tại $B$ của $w$ lấy điểm $A$; lấy điểm $C$ sao cho đoạn thẳng $AC$ cắt $w$ tại hai điểm phân biệt. Đường tròn $w’$ tiếp xúc với $AC$ tại $C$, tiếp xúc với $w$ tại $D$ sao cho $D$ khác phía $B$ đối với $AC$. Chứng minh tâm đường tròn ngoại tiếp tam giác $BCD$ thuộc đường tròn ngoại tiếp tam giác $ABC$.

Giải

  •  Vẽ tiếp tuyến chung tại $D$ của $w$ và $w’$.
  • Ta có $\angle BDC = \angle BDy + \angle yDC = \angle 180^o – \angle xDB$ $+ DCH + \angle 180^\circ – \angle ACD + \angle DCH$  $= \angle BAC + \angle AHB +\angle DCH = \angle BAC + 180^\circ – \angle BDC$.
  • Suy ra $2 \angle BDC = 180^\circ + \angle BAC$. (1)
  • Mặt khác $\angle BTC = 2 (180^\circ – \angle BDC)$, suy ra $2 \angle BDC = 360^\circ – \angle BTC$.(2)
  • Từ (1) và (2) ta có $\angle BAC + \angle BTC = 180^\circ$, vậy tứ giác $ABTC$ nội tiếp.

Ví dụ 3. Tiếp tuyến của đường tròn $(O)$ tại $A$ và $B$ cắt nhau tại điểm $P$. Trên cung nhỏ $AB$ lấy điểm $C$ sao cho $CAB$ khác tam giác cân. Các đường thẳng $CA$ và $BP$ cắt nhau tại $D$, $BC$ và $AP$ cắt nhau tại $E$. Chứng minh rằng tâm đường tròn ngoại tiếp các tam giác $ACE, BCD$ và $OPC$ thẳng hàng.

Giải

  • Gọi $Q$ là giao của $(ACE)$ và $BCD$ ($Q$ khác $C$).Ta có $\angle BDQ = \angle BCQ = \angle QAE$. Suy ra $AQDP$ nội tiếp. Tương tự thì $BQEP$ nội tiếp.
  •  Khi đó $\angle PQC = \angle EQC – \angle EQP = \angle PAC – \angle PBE = \dfrac{1}{2}(\angle AOC – \angle BOC) = \angle POQ$.
  • Vậy tứ giác $OPCQ$ nội tiếp.
  • Từ đó ta có tâm các đường tròn $(ACE), (BCD), (OPC)$ thẳng hàng.

Ví dụ 4. Cho đường tròn $(O)$ và điểm $P$ nằm ngoài $(O)$. Từ $P$ vẽ các tiếp tuyến $PA$ và $PB$ đến $(O)$ với các tiếp điểm $A, B$. Trên tia đối của tia $BP$ lấy điểm $M$. Đường tròn ngoại tiếp tam giác $APM$ cắt $(O)$ tại điểm thứ hai là $D$. Gọi $H$ là hình chiếu của $B$ trên $AM$. Chứng minh rằng $\angle HDM = 2\angle AMP$.

Giải

  • Gọi $E$ là giao điểm của $MD$ và $(O)$, $K$ là giao điểm của $AM$ và $OB$.
  • $\angle xAE = \angle ADE = \angle APM$. Suy ra $AE\parallel PM$, suy ra $\angle EAM = \angle AMP$. (1)
  • Ta có $MD\cdot ME = MB^2 = MH\cdot MK$. Suy ra $DHKE$ nội tiếp. Do đó $\angle HDM = \angle HKE = 2\angle EAM$. (2)
  • Từ (1) và (2) ta có $\angle HDM = 2\angle AMP$.

Điểm thuộc đường cố định (Phần 1)

Đây là phần thuận của bài toán quỹ tích, một dạng toán khó và rất rộng. Trong bài viết nhỏ này tôi xin trình bày một số bước để giải bài toán và một số ví dụ minh họa.

Điểm thuộc đường cố định, thì có thể thuộc đường thẳng hoặc đường tròn, đôi khi giới hạn trong đoạn thẳng hoặc cung tròn. Do đó ta cần trang bị một số kiến thức cơ bản về quỹ tích một số đường hay gặp:

Quỹ tích là đường thẳng.

  1. Quỹ tích các đường thẳng cách đều hai điểm là đường trung trực.
  2. Quỹ tích cách đều hai cạnh của một góc là phân giác của góc đó.
  3. Quỹ tích các điểm cách một đường thẳng một khoảng cho trước là hai đường thẳng song song với đường thẳng đó và cách đường thẳng đó một khoảng đã cho.
  4. Điểm thuộc đường thẳng qua hai điểm cố định, qua một điểm cố định vuông góc hoặc song song với một đường cố định…

Trong một số trường hợp ta chỉ cần chứng minh điểm thuộc đường cố định nào đó, ta lại quy về việc chứng minh ba điểm thẳng hàng.

Ta biết được điểm thuộc đường thẳng hay đường tròn thường ta phải dự đoán bằng cách cho 3 trường hợp phân biệt, trong đó có các trường hợp đặc biệt. Nếu không vẽ thêm hình thì đòi hỏi người làm toán phải có trực giác và cảm nhận hình học tốt. Sau khi dự đoán được thì ta dùng các kiến thức đã biết để tìm lời giải.

Sau đây ta xem một vài ví dụ sau.

Ví dụ 1. Cho đường tròn tâm $O$ đường kính $AB = 2R$. $CD$ là đường kính thay đổi, $AC, AD$ cắt tiếp tuyến tại $B$ của $(O)$ tại các điểm $P, Q$. Chứng minh rằng $CDQP$ nội tiếp và tâm đường tròn ngoại tiếp của tứ giác thuộc một đường cố định.

Gợi ý

Bước dự đoán, ta có thể vẽ hình chính xác cho $CD$ thay đổi rồi dựng điểm $I$, khi vẽ hình chích xác ta xác định được các điểm $I$ sẽ cùng thuộc một đường thẳng.

Đến lúc này, ta hãy liên hệ đường thẳng mà ta phát hiện với các yếu tố có trên hình đó là $O$, đường tròn $(O)$, $AB$ và tiếp tuyến tại $B$.

Nếu phát hiện được đường thẳng đó song song với tiếp tuyến tại $B$ thì ta hãy liên hệ với các quỹ tích hay gặp để tìm ra tính chất.

  • Ta có $\angle ACD = \angle ABD  = \angle AQP$, suy ra $BPCQ$ nội tiếp.
  • Gọi $I$ là tâm đường tròn ngoại tiếp tứ giác. Ta có $IM \bot PQ, IO \bot CD$.
  • Mặt khác, ta có $AM \bot CD, AO \bot PQ$.
  • Khi đó $IM ||AO, IO ||AM$, suy ra $AOIM$ là hình bình hành. Suy ra $IM = AO$ không đổi.
  • Hơn nữa $IM \bot PQ$ và $I, A$ khác phía đối với $PQ$ do đó $I$ thuộc đường thẳng song song với $PQ$ và cách $PQ$ một khoảng bằng bán kính và khác phía $A$ đối với $PQ$.

Ví dụ 2. Cho đường tròn $(O)$ và điểm $A$ nằm ngoài đường tròn, một cát $d$ tuyến qua $A$ cắt $(O)$ tại hai điểm $C, D$. Tiếp tuyến tại $C, D$ cắt nhau tại $P$, chứng minh $P$ luôn thuộc một đường thẳng cố định khi $d$ thay đổi và luôn qua $A$.

Gợi ý

Chỉ cần vẽ hình chính xác ta có thể xác định ngay rằng $P$ thuộc một đường thẳng vuông góc với $AO$, như nhận xét trên, để chứng minh đường thẳng này cố định ta chỉ cần chứng minh nó đi qua một điểm cố định nào đó, việc này dễ dàng khi có thể chứng minh điểm đó thuộc $OA$. Từ đó có cách giải sau:

Gọi $H$ là hình chiếu của $P$ trên $AO$. Ta chứng minh $H$ cố đinh. Gọi $I$ là giao điểm của $OP$ và $CD$.

Ta có $OI.OP = OC^2$ không đổi.

$\triangle OPH \backsim OIA$, suy ra $OH.OA = OI.OP = OC^2$ không đổi. Mà $O, A$ cố định, suy ra $H$ có định.

Do đó $P$ thuộc đường thẳng vuông góc với $OA$ tại $H$ cố định.

Ví dụ 3. (PTNK 2004) Cho đường tròn tâm $O$ bán kính $R$ và điểm $A$ nằm ngoài đường tròn. Một đường thẳng thay đổi qua $A$ cắt $(O)$ tại $B, C$. Chứng minh rằng tâm đường tròn ngoại tiếp tam giác $OBC$ luôn thuộc một đường thẳng cố định.

Gợi ý

Đây là một bài toán khó, nhưng cách giải của nó cũng là kinh nghiệm cho những bài toán khác.

Nhận thấy rằng đường tròn ngoại tiếp tam giác $OBC$ đi qua một điểm cố định là $O$, khi đó để chứng minh tâm $I$ của đường tròn này thuộc một đường thẳng cố định, một cách suy nghĩ tự nhiên là cần chứng minh thêm nó đi qua một điểm cố định khác, khi đó sẽ nằm trên đường trung trực của đoạn thẳng nối $O$ và điểm kia.

Nếu vẽ hình chính xác, ta có thể dự đoán được đường thẳng đó vuông góc với đường $OA$ cố định, khi đó ta có thể nghĩ đến cách như ví dụ 2, vẽ $OH \bot OA$ và chứng minh $OH$ không đổi.

Nói chung tùy cách suy nghĩ ta có thể đi tìm lời giải.

  • Gọi $D$ là giao điểm của $AO$ và $(OBC)$.
  • Ta có $AD.AO = AB.AC = AH^2 = OA^2 – R^2$ không đổi, suy ra $D$ cố định.
  • Do đó tâm $I$ của $(OBC)$ thuộc đường trung trực của đoạn $OD$.

 

Ví dụ 4. Cho tam giác $ABC$, tâm ngoại tiếp là $(O)$. Một đường tròn thay đổi qua $A, O$ cắt các cạnh $AB, AC$ tại $D, E$.

a. Chứng minh rằng hình chiếu của $O$ trên $DE$ thuộc một đường thẳng cố định.

b. Chứng minh rằng trực tâm tam giác $ODE$ thuộc một đường thẳng cố định.

Gợi ý

Gọi $K$ là hình chiếu của $O$ trên $DE$. Ta thấy $ADOE$ nội tiếp và $K$ là hình chiếu $O$ trên $DE$, mô hình quen thuộc, gợi ý cho ta đến một định lý khá quen thuộc.

a.

  • Gọi $M, N$ là hình chiếu của $O$ trên $AB, AC$, ta có $M, N$ là trung điểm của $AB, AC$ nên cố định.
  • Theo định lý Simson thì $M, K, N$ thẳng hàng, hay $K$ thuộc đường thẳng $MN$ cố định.

b.

Nếu vẽ hình chính xác, ta có thể dựđoán được trực tâm $H$ của tam giác $ODE$ thuộc đường thẳng $BC$ cố định, do đó ta chỉ cần chứng minh $B, H, C$ thẳng hàng, ta lại quay về việc chứng minh 3 điểm thẳng hàng.

  • Ta có $\angle OHD = \angle OED = \angle OAD = \angle OBA$, suy ra $ODBH$ nội tiếp.
  • Tương tự ta có $OECH$ nội tiếp.
  • Khi đó $\angle OHB = \angle ODA = \angle OEC = 180^\circ – \angle OHC$. Suy ra $B, H, C$ thẳng hàng.
  • Vậy $H$ thuộc đường thẳng $BC$ cố định.

Bài tập rèn luyện.

  1. Cho đoạn thẳng $AB$ và điểm $M$ thỏa $MA^2 – MB^2 = k$ không đổi. Chứng minh rằng $M$ thuộc một đường thẳng cố định.
  2. Cho tam giác $ABC$, đường tròn thay đổi qua $B, C$ cắt các cạnh $AB, AC$ tại $D, E$. Chứng minh rằng tâm đường tròn ngoại tiếp tam giác $ADE$ luôn thuộc một đường thẳng cố định.
  3. Cho tam giác $ABC$ vuông tại $A$ với $B, C$ cố định. Đường cao $AH$, gọi $D, E$ là hình chiếu của $H$ trên $AB, AC$. Đường tròn đường kính $AH$ cắt đường tròn ngoại tiếp tam giác $ABC$ tại $P$. Gọi $Q$ là giao điểm của $AP$ và $DE$. Chứng minh $Q$ thuộc một đường cố định.
  4. Cho đường tròn $(O)$ cố định và điểm $A$ nằm trong đường tròn, đường thẳng thay đổi qua $A$ cắt $(O)$ tại $B$ và $C$. Gọi $D$ là giao điểm hai tiếp tuyến tại $B$ và $C$ của $(O)$. Chứng minh rằng $D$ thuộc một đường cố định.
  5. Cho tam giác $ABC$ cân tại $A$ nội tiếp đường tròn $(O)$. $D$ là một điểm thay đổi trên cạnh $BC$. Đường tròn $(I)$ qua $D$ và tiếp xúc với cạnh $AB$ tại $B$; đường tròn $(J)$ qua $D$ tiếp xúc với cạnh $AC$ tại $C$. Chứng minh rằng trung điểm của $IJ$ luôn thuộc một đường cố định.
  6. Cho hình chữ nhật $ABCD$. Gọi $H$ là hình chiếu vuông góc của $A$ trên $BD$. $M$ là điểm thay đổi trên đoạn $BH$. Đường tròn ngoại tiếp tam giác $ADM$ cắt $CD$ tại điểm $N$. Chứng minh rằng trung điểm của $MN$ luôn thuộc một đường thẳng cố định.